71
GRAN COMPENDIO DE CIRUGÍA Manejo Inicial del Paciente con Trauma 1. Paciente de 24 años de edad quien al dejar una fiesta colisiona contra otro vehículo frontalmente a más o menos 120 km/h. Estaba ingiriendo licor. Durante el transporte se mantuvo hipotenso 80 mmHg a la palpación. Sus signos vitales al llegar al cuarto de urgencias Pa:110/88, frecuencia cardiaca en 120 X, frecuencia respiratoria 32 X y temperatura 35,6; se palpa frío, diaforético y responde frases incomprensibles al estimularlo. Presenta deformidad de la extremidad inferior con exposición de hueso y sangrado activo. La primera prioridad de este paciente es: a. Radiografía lateral de columna cervical b. Intubación orotraqueal c. Controlar la hemorragia de la extremidad inferior d. Monograma abdominal de urgencias 2. Después de la resucitación la frecuencia cardiaca se encuentra en 100 X, su piel está caliente y su presión es ahora de 118/90 mmHg. Su GCS es de 8. En la placa de tórax se observa un ensanchamiento del mediastino. El USG FAST no se observa líquido en cavidad abdominal. Todo lo siguiente se debe realizar en este paciente, excepto: a. Consulta a neurocirugía b. Celiotomía c. Tomografía de tórax d. Consulta a ortopedia 3. Paciente de 30 años quien es atropellado por un autobús. Ella fue intubada en la escena y se le indicaron líquidos intavenosos. Al arribar al cuarto de urgencias su GSC es de 8, pulso 120 X y su presión arterial es de 100/70. Los paramédicos le administraron oxígeno por AMBU con reservorio a más o menos 20 respiraciones por minuto. Su oxigenación es de 90%. A la auscultación no se escuchan los ruidos respiratorios del lado izquierdo y no se observan las venas del cuello distendidas o desviación de la tráquea. El tratamiento debe consistir primero en: a. Inserción de una aguja número 14 F en el segundo EII b. Colocación de un tubo 32 F en el quinto EII c. Observación de las marcas del tubo endotraqueal, succionar el tubo y lentamente extraerlo mientras se ausculta d. Colocarlo en decúbito lateral derecho para mejorar la ventilación 4. Un varón de 45 años es traído al servicio de urgencias después de sufrir un accidente automovilístico. El paciente está alerta y orientado, con examen neurológico normal. Su fx respiratoria es de 20 x, pulmones limpios bien ventilados, fx cardiaca en 120 x y presión arterial de 6..mmHg. A la exploración física se nota con distensión, establece una línea intravenosa y el paciente recibe rapidamente 2 L de L/R, sin cambios en la frecuencia del pulso ni en la Pa. El siguiente paso y el más apropiado en su Tx es: a. Radiografía de columna cervical b. Inserción de catéter central c. Laparotomía exploradora d. CAT de abdomen e. Lavado peritoneal 5. Según el comité de trauma del colegio americano de cirugía para decisión del triade de los pacientes, lo más importante a tomar en cuenta es:

Gran Compendio Final Cirugia

Embed Size (px)

DESCRIPTION

preguntas cirugia

Citation preview

Page 1: Gran Compendio Final Cirugia

GRAN COMPENDIO DE CIRUGÍA Manejo Inicial del Paciente con Trauma

1. Paciente de 24 años de edad quien al dejar una fiesta colisiona contra otro vehículo frontalmente a más o

menos 120 km/h. Estaba ingiriendo licor. Durante el transporte se mantuvo hipotenso 80 mmHg a la palpación. Sus signos vitales al llegar al cuarto de urgencias Pa:110/88, frecuencia cardiaca en 120 X, frecuencia respiratoria 32 X y temperatura 35,6; se palpa frío, diaforético y responde frases incomprensibles al estimularlo. Presenta deformidad de la extremidad inferior con exposición de hueso y sangrado activo. La primera prioridad de este paciente es:

a. Radiografía lateral de columna cervical b. Intubación orotraqueal c. Controlar la hemorragia de la extremidad inferior d. Monograma abdominal de urgencias

2. Después de la resucitación la frecuencia cardiaca se encuentra en 100 X, su piel está caliente y su presión es

ahora de 118/90 mmHg. Su GCS es de 8. En la placa de tórax se observa un ensanchamiento del mediastino. El USG FAST no se observa líquido en cavidad abdominal. Todo lo siguiente se debe realizar en este paciente, excepto:

a. Consulta a neurocirugía b. Celiotomía c. Tomografía de tórax d. Consulta a ortopedia

3. Paciente de 30 años quien es atropellado por un autobús. Ella fue intubada en la escena y se le indicaron

líquidos intavenosos. Al arribar al cuarto de urgencias su GSC es de 8, pulso 120 X y su presión arterial es de 100/70. Los paramédicos le administraron oxígeno por AMBU con reservorio a más o menos 20 respiraciones por minuto. Su oxigenación es de 90%. A la auscultación no se escuchan los ruidos respiratorios del lado izquierdo y no se observan las venas del cuello distendidas o desviación de la tráquea. El tratamiento debe consistir primero en:

a. Inserción de una aguja número 14 F en el segundo EII b. Colocación de un tubo 32 F en el quinto EII c. Observación de las marcas del tubo endotraqueal, succionar el tubo y lentamente extraerlo mientras se

ausculta d. Colocarlo en decúbito lateral derecho para mejorar la ventilación

4. Un varón de 45 años es traído al servicio de urgencias después de sufrir un accidente automovilístico. El

paciente está alerta y orientado, con examen neurológico normal. Su fx respiratoria es de 20 x, pulmones limpios bien ventilados, fx cardiaca en 120 x y presión arterial de 6..mmHg. A la exploración física se nota con distensión, establece una línea intravenosa y el paciente recibe rapidamente 2 L de L/R, sin cambios en la frecuencia del pulso ni en la Pa. El siguiente paso y el más apropiado en su Tx es:

a. Radiografía de columna cervical b. Inserción de catéter central c. Laparotomía exploradora d. CAT de abdomen e. Lavado peritoneal 5. Según el comité de trauma del colegio americano de cirugía para decisión del triade de los pacientes, lo más

importante a tomar en cuenta es:

Page 2: Gran Compendio Final Cirugia

a. Sitio anatómico de la lesión b. Estados co-morbidos del paciente c. Mecanismo de la lesión d. Signos fisiológicos anormales 6. La primera causa de mortalidad en pacientes menores de 45 años en occidente es: a. Las enfermedades infecciosas b. Las enfermedades cardiovasculares c. Las enfermedades cardiovasculares d. Las enfermedades traumáticas 7. La valoración inicial del paciente traumatizado, según la guía de soporte vital avanzado en trauma (ATLS) del

colegio americano de cirujanos, se divide en 4 fases: a. Valoración pre hospitalaria, transporte del paciente, cirugía urgente y cuidados intensivos b. Acceso inmediato al sistema, cuidados pre hospitalarios, cuidados hospitalarios y rehabilitación c. Reconocimiento primario, resucitación, transporte y cuidados hospitalarios d. Reconocimiento primario, resucitación inmediata, reconocimiento secundario y tx definitivo 8. Un joven de 28 años sufre accidente al caer de su moto. No tenía casco protector. Al llegar al cuarto de

urgencias llega en severo distres respiratorio, Pa 80/50, cianosis, con sangrado protuso por la nariz y fractura expuesta del fémur. Los ruidos ventilatorios se encuentran disminuídos del lado derecho. El Tx prioritario es:

a. Control de la hemorragia con empaque nasal anterior y posterior b. Rx lateral del cuello c. Intubación endotraqueal con inmovilización neutra del cuello d. Toracotomía cerrada derecha e. Canalizar 2 venas y cruce de sangre 9. Al paciente anterior se le suministran 3L de L/R en 10 minutos y continúa hipotenso, usted cree que necesita

sangre inmediatamente, usted transfundiría: a. Sangre tipo específico cruzada b. Sangre O negativo c. Sangre O positivo d. Sangre tipo específico sin cruzar e. Ninguna de las anteriores 10. Después de una laparotomía por herida de bala un hombre de 20 años tiene Hb 8.5g Pa 100/60 Fc 76x usted

: a. Le transfunde 1 u de GRE b. Le transfunde de 1 u de sangre completa c. Repite la prueba de Hg antes de transfundir d. No lo transfunde 11. Una complicación común en pacientes traumatizados que hay que controlar precozmente por ser nociva para

la coagulación y hemodinámica del paciente es : a. Hipoglucemia b. Hiponatremia c. Agitación d. Hipotermia

Page 3: Gran Compendio Final Cirugia

12. Una mujer de 25 años de edad se ve involucrada en un choque de vehículo de motor y sufre una importante

lesión cerrada de cráneo, una contusión pulmonar y fractura de pelvis. Se encuentra sin respuesta a estímulos externos y con ventilación en la unidad de cuidados intensivos. El mejor enfoque para el Tx de las necesidades nutricionales de esta paciente es:

a. Inserción de catéter subclavio e inicio de hiperalimentación central intravenosa b. Esperar para la extubación y mejora de su estado neurológico, que permita instituír ingresos calórico oral c. Instalar tempranamente una sonda nasogástrica o nasoyeyunal para alimentación con fórmula elemental d. Esperar que se resuelva el ileo gastrointestinal, seguido por el inicio tardío de alimentación por sonda

nasogástrica con una fórmula hipercalórica e. Hiperalimentación por vía venosa periférica

13. Paciente de 27 años de edad presenta lesiones en ambas extremidades superiores al caer de una motocicleta

a alta velocidad en la autopista. El aspecto más importante en el manejo de este paciente debe ser: a. Enfriar las partes amputadas o isquémicas b. Dx y Tx de las injurias que amenacen la vida del paciente c. Asegurar una adecuada perfusión d. Dx y reconocer tempranamente el síndrome compartamental

14. En pacientes con traumatismos craneoencefálicos graves, la exploración neurológica se realizará: a. Antes de la valoración de la vía aérea, ventilación y circulación b. Después de la resucitación inicial c. Nunca antes de la llegada a un hospital y debe ser realizada por un neurocirujano d. Después de la cirugía urgente e. Después de la valoración de la vía aérea y de la ventilación y antes de la valoración de la circulación

15. La valoración neurológica durante el reconocimiento primario constará de: a. Valorar nivel de conciencia mediante escala de glassgow b. Valorar nivel de conciencia mediante escala de Glasgow, respuesta pupilar y respuesta motora c. Realizar escáner cerebral d. Valorar nivel de conciencia, reflejos de tronco, miotomas y dermatomas

16. Paciente de 50 años es arrollado por un bus y presenta múltiples fracturas costales en el hemotórax derecho,

presenta movimiento paradójico del tórax y una saturación de oxígeno de 60% con máscara facial a flujo de 10 L/min y FIO2 del 100%. Todo lo siguientes es cierto del trauma torácico cerrado, excepto:

a. Este paciente tiene alta probabilidad de tener una contusión pulmonar b. Es recomendable que se fijen las fracturas costales para mejorar la ventilación c. Se le debe colocar un catéter epidural para el manejo del dolor d. Debe ser enviado a UCI y considerar entubación endotraqueal, luego de realizar toracotomía cerrada

17. El paciente anterior es evaluado por el residente de cirugía y por la severidad de la lesión torácica sospecha

de contusión cardíaca a pesar de que tiene un EKG normal: a. Se le deben tomar enzimas cardiacas seriadas b. Se debe monitorizar el ritmo cardíaco al paciente y se trataran las arritmias c. Se debe tomar EKG cada hora d. Se debe colocar un catéter de Swan Ganz 18. Todas las siguientes consideraciones sugieren una lesión uretral excepto: a. Hematoma escrotal

Page 4: Gran Compendio Final Cirugia

b. Sangre en la luz del recto c. Ausencia de próstata palpable d. Sangre en el meato uretral

19. Después de sufrir un impacto automovilístico llevando puesto es cinturón de seguridad un paciente de 26

años de edad es trasladado al hospital. A la revisión primaria no existe evidencia de lesión grave y solo manifiesta un discreto dolorimiento difuso en todo el abdomen. Los ruidos intestinales son hipoactivos y la matidez hepática parece estar ausente. Las Rx de abdomen revelan aire libre. El paciente debe de:

a. Ser sometido a lavado peritoneal b. Ser sometido a laparatomia de inmediato c. Realizar estudios radiológicos del trato GI con medio de contraste d. Debe ser cuidadosamente observado para descartar evidencia futura

de lesiones intraabdominales

20. La principal causa de acidosis en paciente con traumas múltiples es: a. Hipovolemia b. Hipotermia c. Descarga adrenérgica d. Rabdomiolisis

21. La causa menos probable de depresión del estado de conciencia en una paciente con traumas múltiples es: a. Choque hipovolémico b. Trauma craneal c. Oxigenación disminuida d. Alcohol y drogas 22. Cuál de los siguientes hallazgos debe ser inmediatamente tratado en la evaluación primaria del paciente de

trauma: a. Abdomen distendido y doloroso b. GCS de 8 c. Fx respiratoria de 32 X (Ventilación asistida) d. Pulso de 130 X 23. Cuál de los siguientes elementos es de mayor ayuda en la evaluación de un paciente para prepararlo para

cirugía rutinaria: a. Historia b. Rx de tórax c. Examen Físico d. EKG 24. La mortalidad por trauma es trimodal, el segundo pico corresponde a la mortalidad precoz (4-6 horas) tras el

accidente, habitualmente en el ingreso hospitalario siendo la causa más frecuente: a. El traumatismo craneoncefálico y/o exanguinación b. Obstrucción de la vía aérea y neumotórax a tensión c. Traumatismo abdominal y /o pélvico d. Rotura de aorta y /o cardíaca e. Ninguna de las anteriores

25. La lesión de la columna cervical:

Page 5: Gran Compendio Final Cirugia

a. Es excluida por un examen neurológico normal b. Puede ser detectada al flexionar y extender cuidadosamente el cuello en la evaluación inicial c. Puede ser excluida al tomarle una placa lateral de la columna cervical d. Puede manifestarse primariamente luego de mover el cuello sin ningún cuidado

26. Pareo

a. Laparotomía exploradora A

Paciente con herida por arma de fuego en abdomen y dolor Abdominal

b. US FAST D Paciente de 65 años hemodinamicamente estable con dolor Abdominal

c. Laparoscopía Dx B Paciente hipotenso luego de accidente automovilistico sin dolor Abdominal

d. CAT

Paciente de 20 años con trauma abdominal cerrado, hemodinamicamente estable, con USG FAST positivo porlíquido y lesión esplénica

e. Angiografía f. Lavado peritoneal

27. La intubación naso-traqueal: a. Se debe realizar en pacientes inconscientes que no presenten lesión de columna cervical. b. Se debe realizar en pacientes en que se sospeche lesión cervical y que estén inconscientes. c. Esta contraindicado en pacientes que respiren espontáneamente. d. Se debe realizar en pacientes conscientes con sospecha de lesión cervical.

28. Varios minutos después de haberse caído desde una ventana, un niño de 9 años es traído por sus padres al

departamento de emergencias. El niño esta sangrando profusamente a través de una de herida de 10 cm en su muslo derecho el tratamiento inmediato de la herida debería consistir de:

a. Aplicación de torniquete. b. Colocación de anestesia y punto hemostáticos c. Presión directa sobre la herida d. Aplicación de un apósito de gas estéril. 29. Paciente de 67 años resulta severamente lesionado al colisionar su automóvil contra objeto fijo, no tenía

colocado el cinturón de seguridad. Presenta deformidad de la pelvis y el fémur derecho se encuentra fracturado. La extracción del vehículo fue difícil y dura aproximadamente 30 minutos. Una infusión de lactato ringer fue iniciada en la escena del accidente hasta su llegada al cuarto de urgencias. Su presión arterial se eleva de 70/40 mm Hg a 130/80 mm Hg. A la hora de la resucitación el paciente se encuentra hemodinámicamente estable pero a través de la sonda urinaria no presenta diuresis. El paso inicial para el tratamiento de esta anuria debe incluir:

a. Furosemida 100 mg en bolo. b. Manitol 12,5 gr. En bolo. c. Lactato Ringer a chorro. d. Tomografía axial computarizada de pelvis y abdomen contrastado. e. Ninguna de las anteriores.

Page 6: Gran Compendio Final Cirugia

30. Un hombre de 39 de edad es admitido al cuarto de urgencias luego de haber sufrido una colisión es su

automóvil. El paciente se encuentra cianótico, tiene insuficiencia respiratoria y una escala de Glasgow de 5/15. No hay trauma facial significante; su traque se encuentra en la línea media; y tiene desviación crónica del tabique nasal que impide la intubación naso traqueal. La abundante barba del paciente dificulta la fijación de la máscara de oxígeno. El siguiente paso más apropiado es:

a) Realizar una cricotiroidotomía quirúrgica. b) Forzar un naso traqueal a través del tabique nasal. c) Intentar una intubación oro traqueal usando a 2 personas para mantener la columna alineada. d) Ventilarlo con una mascarilla con válvula y reservorio hasta que se descarte una lesión de la columna

cervical.

Trauma Cervical y Traqueal 31. Paciente de 25 años quién recibe herida por arma blanca en zona II del cuello, se encuentra

hemodinamicamente estable, no disfonía o disfagia. Todos los siguientes estudios se encuentran indicados, excepto:

a. Esofagoscopía Flexible b. Broncoscopía c. Arteriografía d. CAT de cuello 32. El Tx inicial de un trauma laríngeo severo es: a. Traqueotomía b. Cricotiroidotomia por punción c. Intubación orotraqueal d. Oxígeno por cánula nasal 33. Todos los siguientes son factores de riesgos asociados al desarrollo de malignidades de cabeza y cuello,

excepto: a. Tabaquismo b. Ingesta de agua sin flùor c. Ingestión de alcohol d. Exposición a radiación e. Ninguna de las anteriores 34. Un niño de 10 años de edad se presenta al cuarto de urgencias luego de ingerir una bateria de una

calculadora. La rx de tórax demuestra que la bateria se encuentra en el esófago. Cual es el mejor manejo del paciente:

a. Observación b. Endoscopia c. Darle un vaso de leche a tomar d. Administración de IV de glucagón e. Administración sublingual de nifedipina

35. Cual de los siguientes pacientes debe ser transportado prioritariamente al hospital:

Page 7: Gran Compendio Final Cirugia

a. Paciente de 50 años de edad que cayó de una altura de 8 pies con fractura de cadera y signos vitales normales.

b. Paciente de 15 años con trauma craneal cerrado con Glasgow de 13. c. Paciente de 23 años, mujer quien fue victima de asalto, con herida cortante en la espalda, con signos vitales

normales y sin distress respiratorio. d. Paciente de 3 años de edad. Quien era pasajero y el auto en que viajaba colisionó contra objeto fijo en la

autopista con signos vitales normales, con signos de contusión a nivel de la pared abdominal.

36. ¿Cuál de los siguientes enunciados en falso a cerca de los traumas craneoencefálicos? a. La mayoría de los muertos de los accidentes de autos son debidas a lesiones craneales. b. Una rápida y completa evaluación neurológica forma parte de la evaluación inicial de trauma. c. Optimizar la oxigenación arterial es parte fundamental de la terapia inicial de trauma. d. En pacientes hipovolémicos la resucitación con líquidos debe ser muy lenta para no aumentar el edema

cerebral.

37. Paciente de 25 años quien recibe herida por arma Blanca en zona II del cuello, se encuentra hemodinamicamente estable, no disfonía o disfagia. Todos los siguientes estudios se encuentran indicados, excepto:

a. Esofagoscopia flexible b. Broncoscopía c. Ateriografía d. Tomografía computarizada el cuello Trauma Torácico 38. Paciente de 19 años de edad se encuentra hemodinamicamente estable, alerta y bien orientado luego de

haber recibido una herida por arma blanca en la región toraco abdominal izquierda. La herida se encuentra por debajo del pezón, por arriba del márgen costal y en la línea medio clavicular. La radiografía de tórax revela un neumotórax de 30%. Después de la evaluación inicial es completada, el más apropiado Tx es:

a. Exploración local de la herida b. Colocación de tubo pleural y laparatomía c. Dejar en observación y tomar una radiografía control en 6 horas d. Pericardiocentesis

39. Después de que ocurre una lesión torácica la primera maniobra para mejorar la oxigenación es: a. Intubar al paciente b. Evaluación de los gases arteriales c. Administración de oxígeno suplementario d. Obtener una Rx lateral de la columna cervical

40. Paciente de 18 años de edad recibe herida por arma de fuego en hemitórax izquierdo presenta Pa 80/50, Fc

130 x y las venas yugulares distendidas. El tratamiento inicial de este paciente puede consistir en todo lo siguiente, excepto:

a. L/R 2L a chorro b. Administrar oxígeno suplementario con una mascara facial con reservorio

Page 8: Gran Compendio Final Cirugia

c. Toracotomía cerrada d. Toracotomía abierta y pinzamiento de la aorta torácica

41. Todas las siguientes complicaciones del IAM son indicaciones para cirugía, excepto: a. Extrasítoles ventriculares prematuras b. Aneurisma ventricular c. Ruptura ventricular d. Ruptura del septum interventricular 42. Paciente de 18 años de edad recibe herida por arma de fuego en hemitórax izquierdo presenta Pa 80/50, Fc

130 x y las venas yugulares distendidas. El tratamiento inicial de este paciente puede consistir en todo lo siguiente, excepto:

a. L/R 2L a chorro b. Pericardiocentesis c. Toracotomía cerrada

d. Lavado Peritoneal 43. Un varón de 26 años de edad fue atrapado bajo una grúa en una construcción. Después de una prolongada liberación, es traído al servicio de urgencias, inmovilizado en una tabla posterior y recibiendo oxígeno al 100% por máscara. Está alerta y se queja de dolor en el pecho. Tiene saturación de oxígeno al 90%, fX respiratoria de 35 X, fX cardíaca en 120 X y Pa en 85/60. La tráquea está desviada a la derecha, hay crepitación a la palpación del tórax y disminución de los ruidos respiratorios en el hemitórax izquierdo. Cuál es el siguiente paso y el más apropiado en la valoración inicial y el Tx en este paciente: a. Sentar al paciente bien erguido para mejorar el esfuerzo ventilatorio y administrar analgésicos para aliviar las

molestias relacionadas b. Intubar inmediatamente y ventilación asistida c. Tomar Rayos X de tórax portátil d. Descompresión del hemitórax izquierdo con aguja, seguido de la inserción de un tubo de toracostomía e. Toracotomía en el servicio de urgencias 44. Todo es cierto acerca del trauma cardiaco penetrante excepto: a. El EKG muestra cambios inespecíficos b. La triada de Beck se observa en menos del 40% de los pacientes con heridas cardiacas c. Las heridas por arma de fuego tienen una mayor mortalidad d. La presión venosa central es de ayuda para descartar la lesión e. Las enzimas cardiacas están elevadas

45. Paciente de 25 años quien recibe HPAF en región de hipocondrio izquierdo a nivel del 5 EI y otra en región

occipital de la cabeza. El paciente está intubado, Glasgow de 3 Pa 90/60 Fc 100 X Fr 28 X Rx de tórax hay opacidad en todo el hemitórax izquierdo, no se observa el proyectil. El próximo paso a seguir en este paciente será:

a. Envíar inmediatamente CAT cerebral b. Realizar toracotomía cerrada c. Transfundir inmediatamente sangre d. Colocar tubo nasogástrico y sonda urinario

Page 9: Gran Compendio Final Cirugia

46. Paciente de 18 de edad recibe herida por arma de fuego en hemitorax derecho.  Presenta  presión  de  80/40 mm  Hg,  frecuencia  cardíaca  130  x’,  frecuencia  respiratoria  28  x´  GCS 14/15. Al examen físico se encuentra intranquilo diciendo que se va a morir. Los ruidos respiratorios se encuentran abolidos del lado derecho y los ruidos cardíacos están presentes, además presentan ingurgitación yugular bilateral. La prioridad en la atención de este paciente debe enfocarse en: a. La vía aérea b. La ventilación c. La circulación d. El estado neurológico

47. La primera medida a tomar sería: a. Intubación endotraqueal inmediata b. Pericardiocentesis. c. Canalizar 2 venas periféricas y administrar 2 L de Lactato Ringer a chorro. d. Colocar tubo pleural derecho

48. Con respecto a la contusión miocárdica causadas luego de un trauma toráxico, todas son ciertas excepto: a. Son diagnosticadas por estudios cardíacos invasivos. b. Ocurren en menos del 5% de los casos. c. La medición de enzimas cardíacos no ayuda en el diagnóstico. d. El tratamiento es conservador.

49. Después de un impacto vehicular contra una columna de cemento de un puente, un hombre de 25 años es

trasladado al hospital. El hombre está intoxicado tiene un nivel de coma de Glasgow de 13 y se queja de dolor abdominal. Al momento de ser admitido en el hospital, su presión sanguínea era de 80 sistólica a la palpación, pero con la administración de fluidos intravenosos, está rápidamente aumentó a 110 /70 . La frecuencia cardíaca es ahora de 120 palpitaciones por minuto. Las radiografías del tórax muestran pérdida del botón aórtico, ensanchamiento del mediastino, no tiene fracturas costales o hemoneumotorax. El diagnóstico más probable sería:

a. Hemonotórax masivo b. Neumotórax a tensión c. Ruptura de Aorta d. Taponamiento Cardíaco.

50. Un hombre de 55 años fue arrollado por un auto. En el cuarto de urgencias, después de 2 litros de LR. Su presión arterial es de 60/40. la radiografía del tórax es normal. GCS 9/15 el próximo paso a seguir sería:

a. Craneostomía de urgencia b. Laparotomía c. Dos (2) unidades de GRE y reevaluación d. CAT abdominal de urgencias e. CAT cerebral de urgencia.

Page 10: Gran Compendio Final Cirugia

51. Una dama de 40 años estuvo involucrada en un choque frontal con otro vehículo. Su presión arterial es de 100/90 FC 100 xmin. Después de dos (2) litrod de LR la presión arterial es de 115/80 u si FC 96 min. El próximo paso a seguir sería:

a. Laparotomía b. Sangre c. Observación d. CAT abdominal e. Prueba de alcoholemia

52. Un hombre de 28 años recibió una puñalada en el cuadrante inferior izquierdo del abdomen. Su presión

arterial es de 110/60 y su FC es de 80xmin. Refiere dolor en hemiabdomen izquierdo. Todas las siguientes serían medidas de manejo apropiadas. Excepto: (buscar respuestas)

a. Dos litros de LR a chorro b. Laparotomía c. Observación d. Ultrasonido Abdominal e. CAT abdominal

53. Todas las siguientes son ciertas respecto a la TAC de abdomen en el diagnóstico del trauma abdominal

cerrado, excepto: a. Esta indicada en pacientes estables con dolor abdominal b. Está indicada en pacientes estables comatosos sin dolor abdominal c. Está indicada en pacientes parapléjicos d. Es altamente sensible para detectar lesiones hepáticas e. Es altamente sensible para detectar lesiones diafragmáticas

54. El ultrasonido en el trauma abdominal: (buscar respuestas) a. Está indicado en pacientes con Shock b. Es más sensible que el TAC para diagnosticar lesión de viscera huéca. c. Está indicado en pacientes estables con dolor abdominal d. Requiere la administración de contraste endovenoso e. Sólo está indicado cuado no se puede hacer CAT

55. Un hombre de 22 años recibió una puñalada en el hemotórax izquierdo. Su presión arterial es de 60/40 y su

FC 140x min. Las venas yugulares están ingurgitadas a 30º. Su diagnóstico: a. Hemotórax b. Lesión de arteria coronaria c. Fistula broncopleural d. Neumotórax e. Taponamiento cardíaco

56. Un hombre de 30 años recibió una puñalada en el hemotórax izquierdo. Su presión arterial es de 60/40 y su FC es de 120 x min. Los ruidos pulmonares están abolidos en hemotórax izquierdo con hiperresonancia. Ruidos cardíacos normales taquirrítmicos. El próximo paso a seguir sería:

a. Descompresióncon aguja fina

Page 11: Gran Compendio Final Cirugia

b. Toracotomía abierta c. Toracotomía cerrada d. Ultrasonido e. Ventana pericárdica. 57. Un hombre de 22 años de edad presenta una herida de bala en el hemitorax izquierdo. Se coloca un tubo

pleural y se evacuó 700 cc de sangre. Diez minutos después de llegar al Cuarto de Urgencia su presión arterial disminuye a 80/60 y la frecuencia cardíaca aumenta a 136 x’.  El siguiente paso a seguir es: (buscar respuesta b o d)

a. Pinzar el tubo torácico. b. Programar toracotomía de urgencia. c. 2 L de LR a chorro. d. Repetir la evaluación primaria y proceder con el traslado.

58. Un hombre de 60 años de edad sufre una puñalada en el costado posterior derecho. Los testigos del

incidente  indican que el arma era un pequeño cuchillo.   La frecuencia cardíaca del paciente es de 90 x  ‘,    la presión arterial es de 120/72 y la frecuencia respiratoria es de 24x’. la acción más apropiada es:

a. Realizar ultrasonido abdominal. b. Realizar PIV c. Realizar una evaluación física seriada. d. Suturar la herida y enviar al paciente a casa. 59. Un atleta de 22 años de edad recibe una puñalada en el tórax izquierdo a nivel del tercer espacio intercostal

en la línea axilar anterior. Al ser admitida al área de urgencias, se encuentra alerta y despierto. La frecuencia cardíaca es de 100 x’ , la presión arterial es 80/50 y la frecuencia respiratoria es de 20 x’ .  Una radiografía de tórax revela un velamiento completo del hemitorax izquierdo. Se coloca un tubo pleural y se drena 1600 cc de sangre. El siguiente paso en el manejo de esta paciente debe consistir en:

a. Realizar toracoscopia. b. Insertar otro tubo toráxico. c. Prepararía para toracotomía abierta. d. Angiografía y embolización arterial.

Lesión por Quemaduras 60. Durante una explosión una mujer de 52 años de edad sufre quemaduras de sol del 50% de la superficie

corporal. La paciente presenta quemaduras alrededor del tórax y en ambas extremidades superiores. La resucitación inicial es realizada en forma adecuada, la intubación se efectúa por vía nasotraqueal y se inicia una ventilación mecánica. Los niveles de carboxihemoglobina son del 10%, sus análisis de gases arteriales en sangre muestran una PaO2 de 40 mmHg; PaCO2 de 60 mmHg y pH de 7.25. El manejo inmediato de esta paciente debe incluír:

a. Aumentar los líquidos b. Administrar narcóticos endovenosos en pequeñas cantidades c. Reevaluar la vía aérea

Page 12: Gran Compendio Final Cirugia

d. Aplicar presión positiva al final de la expiración

61. La mejor guía de adecuada resucitación de los líquidos en el paciente quemado es: a. Gasto urinario b. Normalización de la frecuencia cardiaca c. 4cc/kg% de superficie corporal quemada en 24 horas (fórmula de Parkland) d. Normalización de la presión venosa central 62. Paciente de 75 años de edad quien sufre quemaduras secundario a explosión de un tanque de gas, la

descripción de las quemaduras es la siguiente, quemadura de primer grado en la cara, toda la extremidad superior derecha y miembro inferior derecho, quemadura de segundo grado en la parte anterior del dorso, miembro inferior izquierdo, quemadura de 3 grado en todo miembro superior izquierdo; el % de la quemadura:

a. 49.5% b. 45% c. 76.55% d. 58.55

63. El tx inicial de este paciente es: a. Oxigeno suplementario b. Traslado inmediatamente a un centro de quemados c. Administrar analgésicos d. Administrar antibióticos e. Realizar faciotomía 64. Paciente de 45 años de edad quien se durmió en su casa con un cigarrillo encendido provocando un incendio.

Llega al hospital de las tablas con quemaduras de 2 grado de 50% de SCT, esputo carbonaceo y dolor de garganta. El cirujano de turno le dice que lo traslade al HST. Lo más importante antes de trasladarlo es:

a. Canalizar 2 venas de grueso calibre y L/R a chorro b. Oxígeno con máscara con reservorio a FIO 40% y flujo de 10 LPM c. Entubar al paciente d. Cubrir las heridas con vendas húmedas y aplicar sulfadizina de plata

65. Varón de 22 años previamente sano de 80 kg fue rescatado de una casa en llamas, después de quedar

atrapado en una pequeña recámara de la casa durante varias horas. Cuando es llevado al servicio de urgencias se le nota combativo y desorientado; sus pulmones están claros en la auscultación bilateral. La fx respiratoria es de 30 X, Pa 100/70 y fX cardiaca de 115 X. Sufrió quemaduras de 2 y 3 grado en 15% de la STC en extremidades inferiores, con lesión circunferencial de grosor completo por debajo de la rodilla derecha. El paso más apropiado en el Tx es:

a. Sedación con midazolam IV b. Morfina IV para analgesia c. Administración de O2 al 100% con mascarilla facial d. Bolo de líquidos IV a razón de 20 cc/kg 66. El líquido apropiado de reanimación sería:

Page 13: Gran Compendio Final Cirugia

a. Dextrosa IV en SSN 0.5 a razón de 150 cc/h b. L/R a tasa de mantenimiento por hora, con bolos de albúmina según sea necesario c. L/R en volumen doble de la tasa de mantenimiento por hora d. 4800 cc de L/R , administrado a 200 cc/h e. Líquidos de mantenimiento IV, además de 4800 cc adicionales de L/R, la mitad administrada en las primeras

8 hrs y el resto en las siguientes 16 hrs

67. Dentro de las primeras horas de ingreso el paciente se queja de dolor en su pie derecho. Se nota tumefacción del pie y del tobillo, disminución del llenado capilar de los dedos del pie derecho y disminución de la sensibilidad. El Tx de elección debería ser:

a. Elevación de la extremidad afectada b. Iniciar antibióticos c. O2 hiperbárico d. Escarotomia 68. Paciente de 24 años de edad quien es encontrada por los bomberos dentro de una casa en llamas, alrededor

de 10 minutos. Le colocan oxigeno por mascara y reservorio a 10 LPM. Al llegar al cuarto de urgencias se encuentra conciente, signos vitales dentro de límites normales. Presenta desaparición de los vellos de la nariz y cejas y tiene cenizas en la nariz. Todo lo siguiente es cierto acerca del tratamiento de esta paciente excepto:

a. En los pacientes quemados se trata inicialmente igual que los otros tipos de trauma. b. Si los pacientes no tienen compromiso de la vía aérea no es necesario intubarlos. c. En pacientes concientes que necesitan tener una vía aérea definitiva deben de ser sedados. d. Pacientes quemados desarrollan un Síndrome de Respuesta Inflamatoria Sistémica.

69. Paciente quien presenta quemaduras de tercer grado circunferenciales en ambos miembros inferiores. Se

queja de adormecimiento y aumento de volumen importante. El tratamiento de inmediato debe ser: a. Elevación de las piernas. b. Escarotomia. c. Diuréticos. d. Analgésicos

70. Manejo inicial de quemaduras de segundo grado y tercer grado: a. Curación y antibióticos b. Llevar al SOP y luego a cuidados intensivos c. Reanimación ABC, Lactato Ringer según cálculo por superficie corporal quemada. d. T/A

71. El tratamiento de los pacientes por quemaduras eléctricas difiere en las quemaduras por fuego con respecto a

la necesidad de: (verificar) a. Fasciotomia. b. Hemodiálisis. c. Amputación. d. Utilización de Oximetria de pulso. e. Resucitación prehospitalaria. 72. La cantidad de fluido intravenoso en 24 horas que debe recibir una mujer de 60 Kg. de 30 años de edad

victima de quemaduras por flama con 80 % de superficie quemada es de:

Page 14: Gran Compendio Final Cirugia

a. 19.2 litros de L/R en 5% de D/A . b. 14.4 litros de L/R. c. 9.6 litros de salina hipertónica. d. 7.2 litros de L/R. 73. Es causa de síndrome de respuesta inflamatoria sistémica:

a. Pancreatitis aguda. b. Quemaduras de segundo grado 30% de SCT. c. Infección por CMV. d. Herida por arma de fuego en abdomen. e. Todas las anteriores.

Clasificación de Heridas e Infecciones Nosocomiales 74. Son señales indirectas de infección post operatoria todas, excepto; a. Aumento en el requerimiento de líquidos b. Ileo adinámico c. Plaquetopenia d. Intolerancia a la glucosa 75. Todo lo siguiente incrementa la tasa de infecciones, en una cirugía mayor; excepto: a. Paciente mayor de 70 años b. Infecciones alejadas del sitio operatorio c. DM controlada con glucosa menor de 200mg/dl d. Ingesta de corticoides

76. Todo lo siguiente incrementa la tasa de infecciones, en una cirugía mayor; excepto: a. Las mordeduras de gatos y monos se debridan y se dejan abiertas si el evento es reciente b. Toda mordedura de animal que no ha recibido atención luego de 24 horas, se debe dejar abierta c. Se deben lavar con abundante agua y jabón d. Las mordeduras tempranas de perro son debridadas y se dejan abiertas 77. La causa más común de fiebre en las primeras 48 horas post operatoria a. Infección de la herida b. Colitis por antibióticos c. Infección asociada a catéteres intra-vasculares d. Atelectasias pulmonares 78. La principal causa de la disminución de las infecciones quierugicas se debe a: a. Seguir los preceptos de lister b. Utilización de antibióticos profilácticos c. Dejar las heridas abiertas d. Utilizar ropas estériles e. Irrigar las heridas con desinfectantes 79. La CDC de Atlanta define infección del sitio quirurgico como una infeccion que ocurre en el sitio de insición

dentro de los: a. 30 días post operatorio o dentro de 12 meses si un implsnte protésico a sido colocado

Page 15: Gran Compendio Final Cirugia

80. Un hombre de 30 años fue operado por apendicitis aguda perforada con cierre primario de la herida

quirúrgica. Tres días después tiene T de 39C, está oligúrico e hipotenso, hay flictenas, crepitación y eritema extenso alrededor de la herida quirúrgica. El manejo inmediato después de la resucitación debe ser:

a. Exploración de la herida con limpieza y debridamiento en salón de operaciones b. Laparotomía exploratoria c. Cámara hiperbárica d. Abrir y curar la herida en sala

81. Son las 2 de la mañana y usted es llamado por la enfermera de la sala de cirugía porque el paciente fue

operado a las 8 am de ayer (colecistotomia abierta), se queja de mucho dolor. Su enfoque de Dx y manejo inicial debe ser:

a. Hx clínica y examen físico b. Rx de tórax c. Hb completo d. Urinálisis

82. Una paciente tiene un catéter venoso central desde hace una semana. Hoy cursa con T 38.8 C y se queja de

dolor en el sitio de inserción el cual se aprecia con escasa secreción mucoide. El siguiente paso a seguir sería: a. Retirar el catéter y colocar otro en el mismo sitio b. Retirar el catéter c. Hemocultivo a través del catéter d. Vancomicina IV

83. Un hombre de 62 años se produce una herida cortante en la mano con una botella de cerveza, cuando estaba

de paseo en la playa. Acude a usted para suturarlo y refiere que tiene todas sus vacunas de la niñez y que hace 14 años atrás le inyectaron toxoide tetánico. Cuál seria su decisión acerca de la prevención de infección y tétano:

a. Inmunoglobulina tetánica más toxoide tetánico b. Toxoide tetánico y lavado con agua estéril o solución salina c. Inmunoglobulina tetánica d. Antibióticos más toxoides tetánicos y solución gaudiano e. Antibióticos más inmunoglobulina tetánica 84. Un hombre de 62 años se produce una herida cortante en la mano con una botella de cerveza, cuando estaba

de paseo en la playa. Acude a usted para suturarlo y refiere que tiene todas sus vacunas de la niñez y que hace 14 años atrás le inyectaron toxoide tetánico. Cuál seria su decisión acerca de la prevención de infección y tétano:

a. Inmunoglobulina tetánica más toxoide tetánico b. Toxoide tetánico c. Inmunoglobulina tetánica d. Antibióticos más toxoide tetánico e. Antibióticos más inmunoglobulina tetánica

85. Cada una de las siguientes preguntas escoja una de las siguientes opciones

Page 16: Gran Compendio Final Cirugia

a) Infecciones nosocomiales en las primeras 48 hrs b) Infecciones nosocomiales posteriores a 48 hrs c) Ambas d) Ninguna

C Hiperglicemia A Streptococo Beta hemolítico B Diarrea C Requerimiento aumentado de líquidos C Ileo adinámico

86. Paciente de 50 años de edad quien desde hace 5 días presenta dolor en epigastrio y en fosa iliaca derecha

asociada a nauseas, vómitos y diarreas liquidas. Ha consultado en tres ocasiones, al cuarto de urgencias y lo han tratado con antibióticos y analgésicos sin mejoría. Ahora presenta dolor en fosa iliaca derecha y sensación de masa en esta área y defensa, además de fiebre y leucocitosis con desviación hacia la izquierda. Es llevado al salón de operaciones y se encuentra una apendicitis aguda perforada con absceso en fosa iliaca derecha y pelvis. 1. Según la clasificación de las heridas este paciente tiene una herida:

a- Limpia. b- Limpia contaminada. c- Contaminada. d- Sucia.

87. Los cuidados postoperatorios de este paciente debe incluir excepto: (verificar) a- Antibióticos intravenosos y curación de la herida con clorhexidina a 7%. b- Ejercicios respiratorios. c- Analgesia. d- Reposición de líquidos y electrolíticos.

88. Al quinto día postoperatorio el paciente cursa con hipotensión, taquicardia y fiebre. La diuresis es menor de

10 c.c. por hora. Además de distensión abdominal. Se revisan los líquidos administrados y el balance se encuentra positivo. Este paciente se le debe realizar:

a- Serie de abdomen agudo. b- Ultrasonido. c- Tomografía axial de abdomen superior. d- Colon por enema.

89. El manejo de este paciente debe incluir excepto:

a- Utilización de inotropicos b- Colocación de catéter central, para medir presión venosa central. c- Traslado a la unidad de cuidados intensivos. d- Aumentar los líquidos y utilizar diuréticos.

90. Todas las siguientes situaciones aumentan el riesgo de infecciones del sitio quirúrgico luego de una

Herniorrafia electiva excepto: a. Rasurar el sitio operatorio el día anterior a la cirugía. b. Colocar drenajes en la herida. c. No continuar los antibióticos profilácticos más de 24 horas. d. Dejar hematomas en la herida. e. Aproximar fuertemente las suturas sobre la piel.

Page 17: Gran Compendio Final Cirugia

91. Con respecto a la preparación pre-operatoria del paciente:

a. El rasurado debe ser preciso y meticuloso b. En casos electivos se debe lavar al paciente con Clorhexidina. c. La preparación de Yodo es bactericida, fungicida y virucida. d. La tintura de Yodo es un iodoforo altamente efectivo

92. Son señales indirectas de infección post-operatoria todas, excepto:

a. Aumento en el requerimiento de líquidos b. Ileo adinámico c. Hiperkalemia d. Intolerancia a la glucosa

93. Ud. es llamada (o) a las 2 am de la sala de cirugía porque el paciente que se le realizó una colecistectomia

abierta a las 8 am del día anterior se queja de mucho dolor. Su enfoque de Dx y manejo inicial incluye todas las siguientes excepto:

a. Historia Clínica y Examen Físico b. R-X de Tórax c. Hemograma completo d. Urinalisis

94. Todo lo siguiente incrementa la tasa de infecciones, en una cirugía mayor excepto: (verificar o es esa o la E)

a- Paciente mayor de 70 años. b- Infecciones alejadas del sitio operatorio. c- Diabetes mellitus controlada. d- Ingesta de corticoides. e- No hay excepto, todas aumentan la tasa de infecciones.

95. Con respecto a los antibióticos profilácticos todas son ciertas excepto: a- Deben iniciarse el día antes de la cirugía. b- Están indicados si las probabilidades de infección son altas. c- Están indicados si las probabilidades de infección son bajas pero una infección podría ser fatal. d- Se requieren niveles séricos óptimos al hacer la incisión. 96. Una paciente tiene un cateter venoso central desde hace 1 semana. Hoy cursa con Tº 38.8 y se queja de dolor

en el sitio de la inserción el cual aprecia con escasa secreción mucoide. El siguiente paso a seguir sería:

a- Retirar el catéter y colocar otro en el mismo sitio b- Retirar el catéter c- Hemocultivo a través del cateter d- Vancomicina IV

97. El cultivo semi-cuantitativo de la punta del catéter venoso central: (verificar) a- No es necesario si hay signos evidentes de infección en el sitio de entrada del catéter. b- Es positivo si se reportan más de 15 colonias. c- Se realiza en medio de tioglicolato. d- También se realiza en casos de sospecha de infección de vías urinaria. 98. Todos los siguientes datos son ciertos acerca del tratamiento de las mordeduras de animales excepto:

a- Se deben lavar con abundate agua y jabón

Page 18: Gran Compendio Final Cirugia

b- Las mordeduras de gatos y monos se debridan y se dejan abiertas si el evento es reciente. c- Toda mordedura de animal que no ha recibido atención luego de 24 horas, se debe dejar abierta d- Las mordeduras tempranas de perro son debridadas y se dejan abiertas.

99. Un hombre de 30 años sufrió heridas varias en las piernas con las cuchillas de una cortadora de césped

mientras trabajaba. (verificar) El manejo inicial debe consistir en:

a- Toxina tetánica y antiglobulina b- Limpieza y sutura en el salón de operaciones c- Morfina 15 mg IV en el Servicio de Urgencias d- T/A

100. Las tres causas más probables de fiebre > 38.5° C, 36 horas después de una colectomía son: a. Atelectasia pulmonar b. Escapes de viscera hueca c. Infección necrotizante de tejidos blandos 101. Un niño es mordido en la mano por un perro. Tiene edema y secreción purulenta. Hay ganglios

epitrocleares y un cordón rojo en el antebrazo. Cual de los siguientes sería el antibiótico indicado: a- Eritromicina b- Ampicilina + sulbactan o Ac. Clavulónico. c- Clindamicina d- Vancomicina

102. Con respecto a los antibióticos profilácticos todas son ciertas excepto:

e- Deben iniciarse el día antes de la cirugía. f- Están indicados si las probabilidades de infección son altas. g- Están indicados si las probabilidades de infección son bajas pero una infección podría ser fatal. h- Se requieren niveles séricos óptimos al hacer la incisión. i- El antibiótico a utilizar se escogerá según la operación a realizar.

103. La causa más común de fiebre en las primeras 48 hrs. Post-operatorias.

a- Infección de la herida b- Colitis por antibióticos c- Infección asociada a catéteres intra-vasculares d- Atelectasias pulmonares

104. Es causa posible de infecciones quirúrgicas después de 72 horas postoperatorias: a- Sinusitis. b- Sepsis asociada al catéter venoso central. c- Absceso intra-abdominal. d- Infección de la herida quirúrgica. e- Todas las anteriores.

105. Un hombre de 50 años tiene fiebre 38.5 y está distendido 5 días después de una apendicectomía por

apendicitis perforada. GB 19,000 con 85% Neutrófilos. El próximo paso a seguir sería: a- Laparotomía b- TAC de abdomen c- Cambio de antibióticos d- Seguir igual

Page 19: Gran Compendio Final Cirugia

106. Un paciente de 40 años fue operado por apendicitis aguda perforada con cierre primario de la herida

quirúrgica. Tres días después tiene temperatura de 39 grados, esta oligurico e hipotenso hay flictenas, crepitación y eritema extenso alrededor de herida quirúrgica.

El manejo inmediato después de la resucitación debe ser: a- Exploración de la herida con limpieza y debridamiento extenso en el SOP b- Laparotomía exploradora c- Cámara hiperbárica d- Abrir y curar la herida en sala e- Se debe realizar luego de exámen endoscópico de la boca, laringe, esófago y tráquea.

107. La CDC (centro de control de infecciones de Atlanta) define una infección del sitio quirúrgico como una

infección que ocurre en el sitio de infección dentro de los _30 días postoperatorios o dentro de 12 meses si un implante protésico ha sido colocado.

Sangrado Digestivo 108. Un hombre de 57 años tiene Hb de 8.5 g después de una hemorragia digestiva alta que ha cesado. Tuvo

un infarto hace 3 meses y está en lista de espera para cirugía de re-vascularización miocárdica. Usted: a. Le transfunde 2 u de GRE b. Le transfunde 2 u de sangre completa c. No le transfunde d. Repite la prueba de Hb antes de transfundir

109. Con respecto al sangrado por varices esofágicas, excepto: a. El propanolol es útil en la hemorragia activa b. El octeortide es el medicamento de elección en el manejo farmacológico inicial c. La ligadura o escleropatía de las varices son parte del manejo rutinario inicial d. TIPS significa shunt porto sistémico intra-hepático transyugular

110. Con respecto al sangrado digestivo bajo: a. Los divertículos son la causa más común b. 50% de los pacientes con diverticulos presentarán episodio de SD bajo secundario a los mismos c. La hemorragia generalmente es de origen venoso d. Si el sangrado es por diverticulos en más del 50% de las veces habrá que operar

111. Con respecto al SD todas son ciertas, excepto: a. Los SD altos son mucho más frecuentes que los SD bajos b. La primera medida en el Dx consiste en pasar un tubo nasogástrico c. Las plaquetas y el plasma fresco son utilizados como parte usual del Tx d. La hematoquesia ocasionalmente puede originarse en TGI alto e. La taquicardia es un signo más temprano que la hipotensión

112. Un hombre de 70 años consulta por hematoquezia. Su Pa 100/60 Fc 110 x. Para establecer el dx todas son ciertas, excepto:

a. Para poder localizar el sitio de sangrado la angiografía necesita una tasa de sangrado superior al centelleo con glóbulos rojos marcados

b. La angiografía da una localización precisa del sitio de sangrado c. La colonoscopia es el primer estudio a realizar

Page 20: Gran Compendio Final Cirugia

d. El scan con glóbulos rojos marcados tiene una severidad del 80%

113. Un Paciente cirrótico acude por hematemesis masiva. Su presión arterial es de 90/70 y su FC 110 x min. Tiene várices esofágicas diagnosticadas con anterioridad. El próximo paso a seguir, sería:

a. Gastroscopia b. Derivación porto cava de urgencia. c. Observación d. Análogo de somatostatina IV. e. Vasopresina IV 114. Una Dama de 85 años, consulta por sangrado transrectal color rojo vino. Su Presión arterial es de 110 / 80

y su FC 90 x min. Todos los siguientes serían conductas apropiadas, excepto: a. Angiografía b. Centelleo con glóbulos rojos marcados. c. Colonoscopia d. Hemostasia quirúrgica. e. Resucitación y estabilización.

115. Todos los siguientes nos hacen sospechar de isquemia intestinal, excepto: a. Arritmias cardíacas b. Fiebre c. Leucocitosis d. Taquicardia e. Vómitos incontrolables

116. Un hombre de 55 años consulta por debilidad y evacuaciones negras. El próximo paso a seguir sería:

(buscar) a. Colonoscopía b. Colon por enema c. CPRE d. Gastroscopía e. Serie esófago gastroduodenal

117. Un hombre de 62 años, se le realizó una endoscopía que mostró una úlcera duodenal con un coágulo

fresco en el fondo. El próximo paso a seguir: a. Cirugía b. Hemostasia endoscópica c. Observación d. Somatostatina IV e. Vasopresina IV

SANGRADO DIGESTIVO BAJO

118. En relación a la hemorragia digestiva baja el uso de centelleo con sulfuro coloidal marcado con tecnecio:

a. es depurado por el sistema reticulo endotelial b. tiene una vida media de 3 minutos

Page 21: Gran Compendio Final Cirugia

c. requiere elocidad de sangrado de 0.5 ml por minuto d. todas las anteriores

119. Paciente de 5 años de edad es traido al cuarto de urgancias del hospital del niño por presentar sangrado transrectal abundante con palidez de tegumento y dolor en FID y flanco derecho. La causa mas frecuente es:

a. diverticulo de meckel b. intolerancia a la leche c. parasitosis intestinal d. angiodisplasia

120. El agente etiológico de la colitis seudomembranosa es:

a- E. Coli. b- Enterobacter fecalis. c- Klebsiella. d- Clostridium difficile. e- Pseudomona.

121. Todos los postulados en relación con el divertículo de Meckel son verdaderos excepto:

a- El divertículo de Meckel aparece en el ileon a mas o menos 90 cm. de la válvula ileocecal. b- El divertículo de Meckel resulta de la falla de obliterarse el ducto vitelino. c- La incidencia del divertículo de Meckel en la población general es del 15%. d- La mucosa gástrica es el tejido ectopico más comúnmente encontrado en el divertículo de Meckel.

122. La complicación más frecuente del divertículo de Meckel es:

a. Sangrado. b. Diverticulitis c. Obstrucción d. Dolor abdominal intermitente

123. Los siguientes genes participan en la secuencia pólipo, adenoma ,carcinoma excepto:

A. DDC(Delete in colorectal cancer) B. K-ras C. HMSH2 D. APC

124. El síndrome de Lynch todas son ciertas excepto: a. Se asocia a múltiples pólipos en el colon b. El cáncer de colon se presenta a edad temprana c. El cáncer es más común en lado derecho d. Suele encontrarse múltiples primarios

e. Todas son ciertas respecto a la enfermedad diverticular del colon excepto: e. Esta asociada a dietas bajas en fibras f. Son pseudo divertículos g. Se presentan principalmente en el colon izquierdo h. Cuando hay sangrado masivo usualmente es del colon izquierdo i. Ninguna de las anteriores

Page 22: Gran Compendio Final Cirugia

125. Un hombre de 68 años acude por dolor abdominal inespecífico en bajo vientre. Ha perdido 20 libras en los últimos 7 meses. Hemoglobina en 9,8 g% y refiere que desde hace 3 meses viene teniendo problemas de constipación . El estudio mas efectivo para Diagnosticar su problema es:

a- SEGD. b- Radiografìa simple de abdomen c- Colonoscopia. d- CAT. e- Antígeno carcinoembrionario.

126. Sus signos vitales son PA 130/80, FC 76 x', con respecto a la anemia de este pte todos son ciertos excepto:

a- No requiere transfusión, si no tiene cardiopatía isquemica. b- El examen de sangre oculta en heces debe ser positivo. c- Las transfusiones de sangre perioperatorias empeoran el pronóstico. d- Debe ser transfundido los más pronto posible para evitar complicaciones.

127. El mejor examen para evaluar la posibilidad de metástasis hepáticas y pélvicas sería CAT

Posterior a la cirugía este pte debe ser seguido con determinaciones séricas de: a- Alfa feto proteína. b- Antígeno carcinoembrionario. c- Determinaciones de C A 19-9. d- Gastrina. 128. Una mujer de 75 años consulta por dolor en Cuadrante inferior izq. (CII) de 48 horas de evolución. T= 38.3

C, GB=15.9 con 90% N, hay defensa y rebote en CII. El próximo paso a seguir sería:

a. Antibióticos IV, hidratación IV. b. Laparotomía c. Colonoscopía d. Colon por enema e. Ninguna de las anteriores

129. Un hombre de 70 años esta siendo tratado por un episodio de diverticulitis desde hace 4 días. Su T es de

38.5° C, G.B. 19.0 con 85%N. El próximo paso a seguir sería: a. Colonoscopía b. Colon por enema c. Laparotomía d. Seguir igual e. Ninguno de los anteriores

Un hombre de 70 años esta siendo tratado por un episodio de diverticulitis desde hace 4 días. Su T es de 38.5° C, G.B. 19.0 con 85%N. El próximo paso a seguir sería:

f. Colonoscopía g. Colon por enema h. TAC abdomino-pelvico i. Seguir igual

Ninguno de los anteriores

Un hombre de 55 años de edad, con antecedentes de diverticulosis, se presenta con cuadro de 2 semanas de evolución con disuria, urgencia y neumaturia. Además, refiere un dolor vago, intermitente, localizado en

Page 23: Gran Compendio Final Cirugia

cuadrante inferior izquierdo del abdomen, de tres meses de evolución y hábitos intestinales irregulares. A la exploración física, tiene sensación de plenitud a la palpación en el cuadrante inferior izquierdo, el cultivo de orina muestra una infección polimicrobiana. En la tomografía por computadora se aprecia una masa inflamatoria en el lado izquierdo de la pelvis. 130. El estudio que más probablemente ayuda para hacer el diagnóstico es:

a- Cistouretrografía de vaciamiento. b- Cistoscopia. c- Enema de bario con contraste aéreo. d- Colonoscopia.

e- Laparoscopia.

131. Después de confirmar el diagnóstico, los pasos iniciales más apropiados en el tratamiento de este paciente incluyen:

a- Antibióticos de amplio espectro e inserción de sonda permanente de vejiga. b- Tratar al enfermo como ambulatorio, antibióticos orales y suavizadores del excremento. c- Líquidos claros, citrato de magnesio y antibióticos orales. d- Laparotomía urgente. e- Colostomia proximal para desfuncionalizar.

132. Un hombre de 55 años consulta por fecaluria, neumaturia e IVU a repetición. En la evaluación de su padecimiento los siguientes pasos a seguir serían: a. Colon por enema y cistoscopía b. TAC y Urocultivo c. Informarle de la necesidad de ser operado d. Todas las anteriores e. Ninguna de las anteriores

133. El tratamiento del vólvulo del sigmoide es: a. Rectosigmoidoscopía inicialmente b. Laparotomía si hay signos de gangrena c. Resección y colostomía si hay que operar d. Todas las anteriores e. Ninguna de las anteriores

134. Son indicaciones de cirugía en la enfermedad de Crohn ¿¿¿excepto?( En un profeta dice excepto y otro no):

a- Perforación. b- Sangrado. c- Obstrucción. d- Debilidad y desnutrición. 135. El agente etiológico de la colitis seudomembranosa es:

a- E. Coli. b- Enterobacter fecalis. c- Klebsiella. d- Clostridium difficile. e-Pseudomona

136. El antibiótico de elección es para tratar la colitis seudomembranosa es:

Page 24: Gran Compendio Final Cirugia

a- Ampicilina. b- Vancomicina. (Luego de Metronidazol) c- Cefoxitina. f- Gentamicina.

SANGRADO DIGESTIVO ALTO 137. En relación al tratamiento de sangrado digestivo alto:

a. el 80% se detiene solo b. el 50% requiere tratamiento de urgencias c. el 45% requiere tratamiento endoscopico d. solamente el 20% se detiene solo.

138. La principal causa de sangrado digestivo al en nuestro medio es:

a. ulcera duodenal cronica activa b. sindrome de mallory –weiss c. varices esofàgicas d. leimioma gastrico

139. el angiograma selectivo determina el sitio de sangramiento agudo cuando este es mayor de a. 0,1 ml por min b. 0,5 ml por min c. 1 ml por min d. 5 ml por minuto

140. En relacion a la hemorragia digestiva alta. Usted tiene una pte que acude cuando con historia de haber vomitado sangre la noche anterior esta hemodinamicamente estable, le realiza una endoscopia digestiva alta de urgencia y el hallazgo endoscópico es una ùlcera duodenal activa con vaso visible, sin sangre digerida o fresca en el estomago usted.

a. lo envia a casa porqu eno hay sangre endoscopica. b. Lo envia a casa con recomendaciones de peptobismol, ranitidina y motronidazol c. Lo hopitaliza pues esta es una lesion de alto riesgo de resangrado d. Lo envia a ca pues al igual que la ulcer e fondo limpio este hallazgo no se asocia con resangrado.

Apendicitis 141. Paciente femenina de 18 años acude al cuarto de urgencias por dolor abdominal de aparición súbita en

bajo vientre de 3 horas de evolución. Al Examen físico el dolor es difuso pero de predominio en Fosa iliaca derecha, sin signos peritoneales GB 9000 con 75% de neutrófilos. Cuál de las siguientes afirmaciones es cierta:

a. La ausencia de signos peritoneales excluye un cuadro quirúrgico b. El hemograma normal excluye un cuadro quirúrgico c. El USG pélvico no está indicado d. La paciente debe ser re-evaluada en 4-5 horas

142. Con respecto al caso anterior todas son ciertas, excepto: a. Se deben iniciar antibióticos inmediatamente b. Se debe solicitar un urinálisis c. La historia menstrual de la paciente es muy importante d. El dolor difuso es típico del dolor viceral

Page 25: Gran Compendio Final Cirugia

143. Una mujer de 18 años tiene historia de 24 horas de evolución de dolor de inicio en mesogastrio y luego irradiado a bajo vientre T 38.5 C, GGB 13.5 con 96% de neutrófilos. FUM hace 3 semanas. Examen fisico hay signos de irritación peritoneal en bajo vientre, principalmente en fosa iliaca derecha. El examen ginecológico es normal, orto negativo. USG pélvico es normal. El próximo paso a seguir sería:

a. Observación por 12 horas b. Apendicectomía c. Laparascopía diagnóstica d. Colon por enema Dolor Abdominal 144. Con respecto al dolor abdominal todas son ciertas; excepto: a. La apendicitis aguda es el Dx quirúrgico más común b. Los mayores de 50 años deben ser estudiados aún cuando desaparezca el dolor c. El dolor parietal es localizado d. El primer síntoma de una obstrucción intestinal es la distensión

145. Hombre de 35 años con dolor en fosa iliaca derecha de 3 días de evolución T 37.5C GB 9.0 con 75% de

neutrófilos, dolor y defensa en fosa ilíaca derecha. Varios episodios similares en las últimas dos semanas. Al examen físico hay aparente masa en fosa iliaca derecha. El próximo paso a seguir sería:

a. US abdominal b. TAC c. Laparotomía d. Laparascopía

146. Después de tres días de antibióticos IV su es de T 38.5 C, GB 22 con 82% neutrófilos. El dolor a

aumentado. El próximo paso a seguir será: a. Nuevo estudio de imagenología b. Colonoscopía c. Colon por enema d. Laparotomía

147. El Tx inicial de la obstrucción intestinal incluye todas, excepto: a. Reposición de líquidos y electrólitos b. Colocación de sondas y monitorización de signos vitales c. Radiografía de tórax y abdomen d. Cirugía inmediata

148. Todas son causas comunes de obstrucción mecánica del colon, excepto: a. vólvulo del sigmoides b. Cáncer de colon c. Diverticulitis sigmoidea d. Síndrome de Ogilvie

149. La causa más común de obstrucción intestinal es: a. Cáncer de colon b. Divertículo de Meckel c. Adherencia post operatorias

Page 26: Gran Compendio Final Cirugia

d. Diverticulitis sigmoideas

150. En la obstrucción intestinal son signos de sufrimiento todos, excepto: a. Leucocitosis b. Fiebre c. Oliguria d. Dolor localizado

151. El síntoma más temprano en la obstrucción intestinal es: a. Distensión abdominal b. Obstipación c. Dolor d. Vómitos

152. Hombre de 65 años acude por hx de 48 horas de evolución caracterizada por dolor cólico, obstinación y

distensión abdominal, tiene una laparatomia previa, la rx muestra niveles hidroaéreos en intestino delgado y gas en colon derecho y transverso, no se ve gas en el colon izquierdo, ni en la ampolla rectal, cual de las siguientes es cierta:

a. La obstrucción del colon por adherencia es común b. La paciente tiene una obstrucción total del intestino delgado c. La obstrucción intestinal de este paciente es poco probable que se resuelva con medidas conservadoras d. Si el ciego mide más de 5 cm es urgente operarlo e. Ninguna de las anteriores

153. El TAC en el manejo del trauma abdominal puede no detectar lesiones de: a. Diafragma e intestino delgado b. Hígado y riñones c. Bazo y páncreas d. No tiene utilidad en el manejo del trauma abdominal e. Detecta todas las lesiones intra abdominales

154. E n el grupo etario entre 20 – 40 años la causa más común de dolor abdominal que requiere cirugía es: a. Obstrucción intestinal. b. Trombosis mesentérica. c. Apendicitis aguda. d. Diverticulitis colónicas.

155. Paciente femenina de 18 años acude al cuarto de urgencias por dolor abdominal de aparición súbita en bajo vientre de 3 hrs de evolución. Al EF el dolor es difuso pero de predominio en FID sin signos peritoneales. GB 9000 con 75% neu.

1. Cuál de las afirmaciones es cierta: a. La ausencia de signos peritoneales excluye un cuadro quirúrgicos. b. El hemograma normal excluye un cuadro quirúrgico. c. El ultrasonido pélvico no esta indicado. d. La paciente debe ser re-evaluada en unas 4-5 hrs.

Page 27: Gran Compendio Final Cirugia

2. Con respecto al caso anterior todas son ciertas excepto: a. Se deben iniciar antibióticos inmediatamente. b. Se debe solicitar un urinálisis. c. La historia menstrual de la paciente es muy importante. d. El dolor difuso es típico del dolor visceral.

156. Con respecto al dolor abdominal todas son ciertas excepto: a. La apendicitis aguda es e DX. Quirúrgico más común. b. Los mayores de 50 años deben ser estudiados aún cuando desaparezca el dolor. c. El dolor parietal es localizado. d. El primer síntoma de una obstrucción intestinal es la distensión.

157. Hombre de 65 años acude por historia de 48 hrs de evolución caracterizada por dolor cólico, constipación

y distensión abdominal. Tiene una laparotomía previa. RX muestras niveles hidro-aéreos en intestino delgado y gas en colon derecho y transverso no se ve gas en colon izq. Ni en la ampolla rectal. Cual de las siguientes es cierta:

a. La obstrucción del colon por adherencias en común

b. El paciente tiene una obstrucción total del intestino delgado

c. La obstrucción intestinal de este paciente es poco probable que se resuelva con medicinas

conservadores

d. Si el ciego mide más de 5 cms es urgente operarlo

e. Ninguna de las anteriores

158. Mujer de 18 años tiene historia de 24 horas de evolución de dolor de inicio en mesogastrio y luego irradiado a bajo vientre. T 38.5 ºC, GB 13.5 con 96% de neutrófilos. FUM hace 3 semanas. (buscar respuesta) a. Observación por 12 horas

b. Apendicectomía

c. Laparoscopía diagnóstica

d. Colon por Enema

e. Ninguno de las anteriores

159. Hombre de 65 años con dolor en FID de 3 días de evolución T 37.5 ºC GB 9.0 con 75% neu. Dolor y defensa

en FID. Varios episodios similares en las últimas dos semanas. Al E.F. hay aparente masa en FID. El próximo paso a seguir sería (buscar respuesta) a. US abdominal

b. TAC

c. Laparotomía

d. Laparoscopía

Page 28: Gran Compendio Final Cirugia

e. Ninguna de las anteriores

160. Después de tres días de antibióticos IV su Tº es de 38.5 ºC GB 22 con 82% neu. El dolor ha aumentado, El

próximo paso a seguir sería: a. Nuevo estudio de imagenología

b. Colonoscopía

c. Colon por enema

d. Laparotomía

e. Ninguna de las anteriores.

161. En el diagnóstico de dolor abdominal, el factor más importante a

considerar es: (buscar respuesta)

Edad del paciente

Estado socioeconómico

a. Ocupación del paciente b. Sexo c. Raza

162. Una dama de 22 años consulta por dolor abdominal de más de 12 horas de evolución. GB 8,000 con 66%

neutrófilos. Hay defensa y rebote en todo el hemiabdomen inferior. El próximo paso a seguir sería: (buscar respuesta)

a. Apendicectomía b. Laparoscopía c. Observación d. Ultrasonido e. TAC abdominal

163. Un hombre de 20 años consulta por dolor abdominal de 12 horas de evolución. Al EF no hay signos de

irritación peritoneal. Sólo dolor leve al a palpación profunda en FID GB 16,000 con 85% de neutrófilos. El diagnóstico más probable.

a. Adenitis mesentérica b. Apendicitis aguda c. Gastroenteritis d. Colecititis e. Pancreatitis

Page 29: Gran Compendio Final Cirugia

164. Un hombre de 70 años, consulta por dolor abdominal, de más o menos 24 horas de evolución. El dolor es tipo cólico. No hay signos de irritación peritoneal, pero si hay dolor leve en fosa ilíaca izquierda. GB 10,5000 N 78% Hgb 11.2 g. Hay escasos niveles hidroaéreos del intestino delgado. Hay historia de pérdida de peso no cuantificada durante los últimos dos meses. El diagnóstico más probable sería:

a. Apendicitis guda b. Diverticulitis aguda c. Isquemia intestinal d. Obstrucción intestinal e. Tiflitis 165. Un hombre de 25 añosconsulta por dolor abdominal tipo cólico de más o menos 6 horas de evolución y

vómitos. Tiene una laparotomía previa hace 1 año por trauma penetrante. No esta distendido, no hay nivles hidroáreos. GB 8,700 con 78% de neutrófilos. El diagnóstico más probable:

a. Apendicitis aguda b. Gastritis c. Litiasis urinaria d. Obstrucción intestinal

Cáncer de Colon 166. Cuál de las siguientes se consideran lesiones premalignas del colon: a. Pólipos inflamatorios b. Pólipos adenomatosos c. Pólipos hiperplásicos d. Pólipos hamartomatosos

167. La causa más común de hipercalcemia es: a. Cáncer b. Hiperparatiroidismo primario c. Insuficiencia renal d. Deshidratación

Page 30: Gran Compendio Final Cirugia

168. Con respecto al Cancer Colorectal todos son ciertas excepto: a. Solo los pólipos adenomatosos tienen potencial de malignización b. Las mutaciones k-ras están implicadas en la transformación de un adenoma en adenocarcinoma. c. La detección y remoción de pólipos no ha logrado reducir la incidencia de cáncer de colon. d. El factor pronóstico mas importante es la presencia de adenopatías metastásicas. e. Las dietas bajas en fibras y altas en grasas aumentan la incidencia de cancer de colon. 169. Con respecto al cáncer colorectal todas son ciertas excepto: a. En 15% de los casos hay una historia familiar positiva. b. En 75% de los casos no hay factores predisponentes identificables. c. Los exámenes de pesquiza se deben iniciar a los 60 años. d. La determinación de sangre oculta anualmente mas sigmoidoscopía flexible c/5 años es un protocolo aceptable de pesquiza. e. Colonoscopía c/10 años es un protocolo aceptable de pesquiza. 170. Con respecto al cancer colorectal todas son ciertas excepto: (a o c) a. El cancer colorectal es sintomático desde muy temprano en su evolución. b. El tenesmo es un síntoma de cancer rectal. c. Los cambios en los hábitos de defecación son comunes en el cancer del colon pero no del recto. d. La anemia es mas frecuente en canceres de colon derecho. e. Casi el 70% de los casos se presentan al alcance del sigmoidoscopio flexible. 171. Con respecto al cancer colorectal todas son ciertas excepto: e. El antígeno carcino-embrionario tiene mas utilidad en el seguimiento que en el tratamiento inicial. f. La quimioterapia adyuvante siempre está indicado en los Estadios III. g. La Quimio-radiación neo adyuvante no mejora la sobrevida en el cancer rectal estadío III. h. En la evaluación pre-operatoria de los pacientes electivos son importantes la TAC y las PFH. i. Si el paciente esta obstruído los examenes pre-operatorios no son necesario

Page 31: Gran Compendio Final Cirugia

172. El cáncer colorectal el gen p53: a. se asocia a mutacion del gen 5q21 b. participa en la muerte celular programada c. sensibiliza las cèlulas a los factores de crecimiento d. es un oncogen estimulador de cancer 173. cual es la localizacion màs frecunte del ca de colonico: a. colon ascendente b. recto c. colon transverso d. colon descendente e. sigmoides 174. mujer anciana admitida por debilidad, asma perdida de peso, y masa palpable abdominal Ella tiene un

carcinoma de recto. El sitio anatomico es: a. recto b. colon sigmoides c. colon izquierdo d. colon transverso e. colon drecho 175. Los pólipos de colon que se consideran potencialmente malignos son: a- Pólipos inflamatorios. b- Pólipos hiperplasicos. c- Pólipos hamartomatosos. d- Pólipos adenomatosos. e- Todos los anteriores 176. En la poliposis adenomatosa familiar el riesgo de cancer de colon es; a. 1% sin tratamiento b. 10% sin tratamiento c. 100% sin tratamiento d. no hay riesgo de cancer de colon

Page 32: Gran Compendio Final Cirugia

177. A un hombre de 65 años se le realizó una colectomía por un Adeno Carcinoma de sigmoides hace 6 días. Actualmente su estado general es bueno pero tiene T de 38.8°C desde hace un día. GB=15.3 con 80% N. Con respecto a la evaluación de la infección post-operatoria de este paciente todas son ciertas excepto:

a. Una TAC de abdomen es el mejor método para investigar la presencia de infecciones de órganos y espacios del sitio operatorio. b. La presencia de infecciones relacionadas a dispositivos IV solo requiere de un cultivo cualitativo de la punta del catéter. c. En este periodo las infecciones de la herida por Streptococos d. La toxina del Clostridium dificcile es la responsable de los síntomas de infecciones entéricas. 178. según la clasificacion de dukes para el cancer de colorectal el tumor que se extiende hasta la muscular, se

refiere al estadio: a. A b. B1 c. B2 d. C e. D 179. cuando el tumor de colon afecta la serosa , según la clasificación de dukes se clasifica como: a. dukes a b. dukes b2 c. dukes c d. dukes d e. ninguna de las anteriores 180. el síntoma más caracteristico de sospecha de cáncer de colon izquierdo: a. obstrucción b. dolor còlico c. anemia d. perdida de peso e. ninguna de las anteriores 181. la prueba mas utilizada para detección temprana de cáncer de colon es: a. rectoscopia b. colon por enema c. sangre en heces d. colonoscopia e. ninguna de las anteriores Anomalías Anorectales

182. El examen más preciso en la evaluación del cáncer de recto es:

a. El tacto rectal b. La tomografía axial computarizada c. El ultrasonido endorectal d. La resonancia magnética nuclear

183. En la fisura anal la tríada de Brodie se caracteriza por:

a. Sangrado, dolor y masa anal b. Fisura anal lateral, prurito e ictericia

Page 33: Gran Compendio Final Cirugia

c. Fisura anal, colgajo cutáneo y papila hipertrófica d. Hemorroides grado III, dolor y papila hipertrófica.

184. Un hombre de 35 años acude con dolor perianal que no le permite sentarse, fiebre y diarrea de 4 días

de evolución. Al E.F. la región perianal es normal excepto por un endurecimiento muy doloroso del lado derecho que hace muy difícil el examen. El manejo debe ser:

a- Incisión y drenaje del absceso perianal b- Baños de asiento por hemorroides internas. c- Antibióticos por su gastroenteritis. d- Laxantes por su fisura anal. a- Ninguna de las anteriores

185. En la fisura anal el dolor post-defecatorio se debe a: a. Apertura de la fisura anal b. Espasmo del esfínter externo c. Espasmo del esfínter interno d. Espasmo del elevador del ano

186. Las causas más comunes de dolor anal son: a. Fístula, hemorroides sangrante y proctalgia fugax b. Abscesos, fístulas y hemorroides internas c. Cáncer de ano, hemorroides y fisuras d. Abscesos, fisuras y trombosis hemorroidal externa

Page 34: Gran Compendio Final Cirugia

187. Las hemorroides internas grado II son: a. Las que sangran sin prolapsarse b. Las que se prolapsan y se reducen manualmente c. Las que se prolapsan y se reducen espontáneamente d. Las que se prolapsan y no se pueden reducir

188. hay una indicación quirurgica clara en pacientes con enfermedad hemorroidal: e. si las hemorroides son grado I f. Si las hemorroides estan trombosadas g. Si la paciente esta embarazada y tiene hemorroides grado III h. Nunca b. Siempre

189. Las hemorroides internas de tercer grado se caracterizan por: a. prolapso hemorroidal que se reduce espontàneamente b. prolapso hemorroidal que se reduce manualmente c. prolapso hemorroidal encarcelado d. ninguna de las anteriores. 190. El Tratamiento de los abscesos anales consiste en :

a. antibioticos b. baños de asciento c. incisión y drenaje d. anti-inflamatorios

191. El tratamiento del absceso anal siempre es drenaje, excepto en:

a. Ancianos b. Inmunosuprimidos c. Pacientes con insuficiencia renal d. Pacientes leucémicos

192. En los abscesos anales los antibióticos están indicados en :

a. diabéticos b. inmunosuprimidos c. leucémicos d. todos los anteriores e. ninguno de los anteriores

193. La complicacion mas severa del abseco anorectal no tratado es:

a. gangrena meleney b. fascitis necrotizante c. mionecrosis d. gangrena de fournier

194. Las fístulas anales se caracterizan por

a. Orificio perianal con descarga de pus intermitente b. Sangrado al evacuar c. Masa pediculada en el ano luego de un absceso d. Eritema perianal asociado a prurito

Page 35: Gran Compendio Final Cirugia

195. Fiebre y dolor perianal son caracterìsticos de: a. hemorroides grado IV b. Abseso perianal c. Fisura anal crònica d. Quiste pilonidal e. oxiuriasis

Cirugía Pediátrica 196. Paciente de 6 meses de edad la madre refiere que se encuentra intranquilo, no quiere la leche, vómitos

verdosos. Al examen físico se encuentra intranquilo e impresiona masa dura en flanco derecho. El dx más probable que tenga este paciente es:

a. Intususcepción b. Plastrón apendicular c. Estenosis pilórica d. Malrotación intestinal 197. Paciente de 2 días de nacido quién refiere la madre, que al llorar presenta masa en la región inguinal y

luego se desaparece. El diagnóstico más probable de este paciente es: a. Hernia Inguinal b. Quiste del cordón c. Hidrocele d. Tumor testicular

Page 36: Gran Compendio Final Cirugia

198. Una niña de 4 años de edad previamente sana, es traída al servicio de urgencias con un cuadro de 24

horas de evolución caracterizado por sangrado rectal y mareo, sin síntomas GI. A la exploración física se encuentra pálida, Fc 140, presenta hipotensión postural. Su abdomen no está distendido ni es doloroso y al examen rectal hay sangre fresca y coágulos en el ámpula. El diagnóstico más probable es:

a. Divertículo de Meckel sangrante b. Pólipo rectal juvenil c. Hemorroides d. Fisura anal e. Intusucepción

199. Paciente de 24 horas de nacido, se observa taquipneico, al examen físico se encuentra un abdomen excavado y el murmullo vesicular disminuido en lado derecho y matidez. El diagnóstico más probable es:

a. Atresia esofágica. b. Ruptura de bula enfisematosa congénita. c. Hernia diafragmática d. Neumotórax por barotrauma.

200. Paciente de 2 años de edad, es traído por su madre al cuarto de urgencias por presentarse irritable con vómitos biliosos, además presenta heces diarreicas rojas con moco y espesas. Al examen físico se palpa una tumoración en fosa y flanco derecho el diagnostico más probable de este paciente es:

a. Mal rotación intestinal b. Plastrón apendicular c. Invaginación intestinal d. Megacolon congénito.

Page 37: Gran Compendio Final Cirugia

Hernias e Hidrocele 201. Paciente de 25 años de edad, se queja que presenta una masa no reducible del testículo derecho al

examen físico no se reduce, hay transiluminación y no se palpa el testículo derecho. El diagnóstico más probable es:

a. Hernia inguinal b. Quiste del cordón c. Hidrocele d. Tumor testicular

202. Un hombre obeso de 58 años acude por presentar una tumoración inguinal dolorosa con signos

inflamatorios leves que no desaparece desde hace 24 horas. Dice que tiene una hernia del mismo lado pero que normalmente se la puede reducir. Usted:

a. Usted le ordena AINES para re-evaluarlo en 24 horas b. Le punciona la masa c. Le dice que necesita cirugía urgente d. Trata de ver si se puede reducir la masa 203. Un hombre obeso de 58 años acude por presentar una tumoración inguinal dolorosa con signos

inflamatorios leves que no desaparece desde hace 24 horas. Dice que tiene una hernia del mismo lado pero que normalmente se la puede reducir. Usted:

a. Usted le ordena AINES para re-evaluarlo en 24 horas b. Le punciona la masa c. Le dice que necesita cirugía urgente d. Trata de ver si se puede reducir la masa e. Le prescribe antibióticos

204. Cuando un órgano forma parte del saco herniario, esta hernia recibe el nombre de: a. Hernia deslizante b. Hernia de Ritcher c. Hernia Encarcelada d. Hernia Inguinal indirecta e. Hernia en pantalón

205. La persistencia del proceso vaginalis muchas veces resulta en: a. Hidrocele del cordón espermático b. Hernia inguinal indirecta c. Hernia inguinal directa d. Hernia Femoral e. A y b f. A,b,c

206. El músculo cremaster que recubre el cordón inguinal es la prolongación de: a. Músculo oblicuo externo o mayor b. Músculo oblicuo interno o menor c. Músculo transverso del abdomen d. Fascia transversales e. Músculo Recto Abdominal

Page 38: Gran Compendio Final Cirugia

207. La tasa más baja de recurrencias en operaciones por hernias inguinales se observa con: a. Herniorrafia tipo Bassini b. Herniorrafia tipo Mc Vay c. Herniorrafia abierta con malla de polipropileno d. Todas tienen tasas de recurrencias similares 208. La hernia inguinal directa: a. Sale por el triángulo de Hasselbach b. Es adquirida c. Está relacionada a esfuerzos físicos d. Todas las anteriores

209. La hernia inguinal indirecta: a. Es congénita b. Sale por el anillo profundo c. Está rodeada por el músculo cremaster d. Todas las anteriores Tiroides y Paratiroides 210. El síntoma más temprano de hipocalcemia post operatoria es: a. Signo de Chvostek b. Signo de trousseau c. Espasmo carpo-pedal d. Parestesias peri-orales

Page 39: Gran Compendio Final Cirugia

211. Son signos de mal pronóstico en un paciente con nódulo tiroideo todos menos: a. Tamaño mayor de 3 cm b. Sexo masculino c. Exposición a radiación d. Dieta baja en yodo e. Edad mayor de 40 años

212. Una mujer de 60 años asintomática tiene un nódulo tiroideo de 3 cm de diámetro. El primer estudio Dx a

realizar debe ser: a. Punción con aguja fina b. T4 c. Centelleo de tiroides d. Ultrasonido de tiroides e. CAT de cuello

213. Paciente de 15 años que presenta masa en la región cervical del cuello, dolorosa y eritematosa, al elevar

la lengua la masa se mueve hacia arriba. . El Dx más probable es: a. Higroma quístico b. Adenopatía c. Neoplasia de tiroides d. Quiste tirogloso

214. Cuando la biopsia de una masa de cuello en pacientes con riesgo de cáncer debe ser realizada: a. Inmediatamente luego de ser descubierta b. Inmediatamente luego de antibióticos c. Cuando no hay factores de malignidad d. Si la masa no está presenta desde la infancia e. Se debe realizar luego de examen endoscopico de la boca, laringe, esófago y tráquea

215. Paciente de 65 años de edad, alcohólico y fumador crónico, además de vómitos post pandriales y pérdida

de peso no cuantificada. Presenta una masa en región supraesternal izquierda del cuello, el manejo más apropiado sería:

a. Antibióticos y anti inflamatorios b. Observación c. Biopsia luego de endoscopia, laringoscopía y broncoscopia d. Biopsia inmediata

Page 40: Gran Compendio Final Cirugia

216. Son complicaciones de las tiroidectomías todas, excepto: a. hipocalcemia b. Lesión del nervio laríngeo recurrente c. Hipofosfatemia d. Lesión del nervio laríngeo superior e. Hemorragia 217. El tumor de parótida benigna más común es el: a. Adenoma pleomórfico b. Lipoma c. Adenocarcinoma d. Linfoma e. Ninguna de las anteriores 218. La causa más común de sodio sérico de 125 mEq/L en un paciente post operado es: a. Déficit de agua b. Exceso de agua c. Déficit de sodio d. Exceso de sodio e. Ninguna de las anteriores 219. La pseudohiponatremia ocurre secundariamente en la siguiente Patología: a. Enfermedades del SNC b. Enfermedad de Addison c. Acidosis tubular renal d. Hiperglicemia e. Hipopituitarismo Una mujer 39 años de edad se presenta al consultorio del médico para la evaluación de un nódulo palpable en el cuello de 2 años de evolución. Con antecedente de enfermedad de Hashimoto diagnosticada hace 5 años, para la que toma hormona tiroidea. La paciente tiene antecedente de radioterapia en tórax de dosis baja por un crecimiento del timo durante la infancia. En la exploración física se palpa un nódulo de 2,5 cm en el lóbulo izquierdo de la tiroides, firme y no doloroso. 220. ¿Cuál de sus antecedentes aumenta el riesgo para cáncer tiroideo?

a- Grupo de edad de 20-40 años. b- Género femenino. c- Radiación de dosis bajas durante la infancia. d- Antecedentes de la enfermedad de Hashimoto. e- Todas las anteriores.

221. ¿Cuál de las siguientes es el mejor paso a seguir.

a- Ultrasonido del cuello. b- Centelleo de tiroides. c- Tomografía de cuello y tórax. d- Aspiración del nódulo con aguja fina. e- Todas las anteriores.

Page 41: Gran Compendio Final Cirugia

222. El hipertiroidismo puede ser causado por las siguientes enfermedades excepto: a. Enfermedad de Graves.

b. Enfermedad de Hashimoto. c- Enfermedad de Plumier. d- Carcinoma medular de tiroides. 223. Un hombre de 55 años tiene un nódulo en el polo inferior del lóbulo derecho de la tiroides. El nódulo

mide 4 cms la PAAF fue reportada como sospechosa. Usted le recomienda.

a- Cirugía b- Iodo radioactivo c- Observación d- Otra punción e- Tratamiento supresivo

224. Un hombre de 55 años tiene un nódulo en el polo inferior del lóbulo derecho de la tiroides. El nódulo

mide 4 cm y la PAAF fue reportada como sospechosa. Usted le recomienda: a. Cirugía b. Yodo radioactivo c. Observación d. Otra punción e. Tratamiento supresivo 225. El procedimiento diagnostico de tamizaje para diferenciar un nódulo tiroideo maligno de uno benigno: a. USG de tiroides. b. Centelleo de tiroides. c. Biopsia por aspiración con aguja fina. d. Prueba de supresión de hormona tiroidea. a. Antibióticos de amplio espectro b. TAC en las primeras 48 horas c. Atención en la unidad de cuidados intensivos d. Laparotomía por necrosis peri-pancreáticas

e. Monitoreo hemodinámica invasivo 226. La ronquera secundaria a carcinoma broncogénico es usualmente debido a invasión del tumor a:

a. Cuerdas vocales b. Nervio Laringeo superior c. Nervio laringeo recurrente d. Laringe

Ulceras 227. Un hombre de 65 años que toma AINES diariamente por dolores articulares acude por historia de

hematemesis, taquicardia y diaforesis. La endoscopía alta revela una úlcera en el bulbo duodenal sin sangrado activo pero con vaso visible. El paso a seguir sería:

a. A sala con infusión de omeprazole b. Vaguectomía más piloroplastía c. Aplicar electrocauterio o clips endoscopicamente al vaso visible d. Balón de compresión intra gástrico e. Lavados gástricos contínuos

Page 42: Gran Compendio Final Cirugia

228. Un sujeto de 55 años de edad asiste al consultorio del médico, se queja de dolor en la parte alta del

abdomen de 2 meses de evolución. El dolor se describe como retorcijón, localizado en hipogastrio, acompañado de náuseas, el dolor se exacerba con los alimentos y pérdida de peso de 10 kg, los últimos 2 meses. Tiene antecedentes de tabaquismo, ingesta de alcohol ocasional, así como Dx de úlcera gástrica benigna. Cuál de las pruebas siguientes es el método más fidedigno para Dx una úlcera gástrica benigna:

a. Serie esofagogastroduodenal con bario b. Endoscopia con fibra óptica superior c. CAT abdominal d. USG endoscópico

229. Es indicación para cirugía en la enfermedad úlcero péptica: a. Las úlceras no curan luego de 12-15 semanas de tx b. Cuando hay recurrencia de la úlcera, a pesar de su tx médico c. Cuando ocurre una complicación d. Todas las anteriores e. Ninguna de las anteriores

Page 43: Gran Compendio Final Cirugia

230. Las úlceras gástricas tipo III se caracterizan por: a. Se encuentran en la curvatura menor b. Están siempre asociadas a una úlcera duodenal c. Son úlceras gástricas prepilóricas d. Cerca de un tercio se encuentran asociadas a la ingestión de aspirinas o AINES e. Ninguna de las anteriores

231. En este paciente se encontró una úlcera gástrica benigna y se instaló el Tx con un inhibidor de la bomba

de protones y 3 esquema de antibióticos para Helycobacter pylori, el paciente regresó al consultorio del médico 3 meses después con la misma sintomatología y en la revaloración se encontró que la úlcera gástrica persistía. Cuál de las siguientes opciones es la mejor para continuar el Tx:

a. Un segundo Tx de inhibidores de la bomba de protones con triple esquema de antibiótico y reevaluación en 2 meses

b. Un Tx de prueba con sucralfalto y reevaluación en 2 meses c. Tx quirúrgico d. Un Tx con prostaglandinas y revaloración en 2 meses

232. Un hombre de 65 años que toma AINES diariamente por dolores articulares acude por hx de

hematemesis, taquicardia y diaforesis. La endoscopia alta revela una úlcera en el bulbo duodenal sin sangrado activo pero con vaso visible, el paso siguiente a seguir será:

a. A sala con infusión de omeprazol b. Vaguectomía más piloroplatía c. Aplicar electrocauterio o clips endoscopicamente al vaso visible d. Balón de compresión intra gástrico e. Lavados gástricos contínuos

233. La primera línea de la terapia de la enfermedad ulcero péptica incluye: a. Vaguectomia + piloroplastia b. Antrectomia c. Determinación de gastrina d. Descartar Helicobacter pylori e. Anti H2

Page 44: Gran Compendio Final Cirugia

Cáncer de Mama 234. Paciente de 60 años con cáncer intraductal mamario, tumor de 2 cm. Sin ganglios palpables en axila, el

mejor Tx a seguir es: a. Mastectomía radical modificada b. Lumpectomía más vaciamiento axilar sin radio Tx post operatorio c. Lumpectomía más biopsia del ganglio centinela con radio Tx post operatorio d. Lumpectomía solamente 235. Una paciente tiene un tumor de 6 cm en la mama. Una biopsia con aguja

Dx carcinoma ductal infiltrante. El mejor Tx inicial a seguir será: a. Quimioterapia pre operatoria b. Mastectomía radical modificada c. Lumpectomía más vaciamiento axilar con radioterapia post operatoria d. Lumpectomía más vaciamiento axilar sin radioterapia post operat e. Ninguna de las anteriores

236. Todo lo siguiente es cierto acerca de cáncer de mama no invasivo, excepto: a. Se define como aquel cáncer de mama que no avanza más allá de la membrana basal b. Puede ser ductal o lobular c. El lobular nunca se palpa d. Ambos tienen buen pronóstico y se observan como microcalcificaciones en la mamografía

237. Paciente de 60 años con cáncer intraductal mamario, tumor de 2 cm, sin ganglios palpables en axila, el

mejor tx a seguir es: a. Mastectomía radical modificada b. Lumpectomía más vaciamiento axilar sin radio tx post operatorio c. Lumpectomía más biopsia del ganglio centinela con radio tx post operatorio d. Lumpectomía solamente e. Ninguna de las anteriores

Page 45: Gran Compendio Final Cirugia

238. Una mujer de 62 años se le ha diagnosticado un carcinoma ductal infiltrante de la mama T1 N0 M0 receptores estrogénicos negativos después de lumpectomía y biopsia del ganglio centinela. El próximo paso a seguir sería.

a. Mastectomía radical modificada b. Quimioterapia c. Radioterapia d. Seguimiento en 3 meses e. Tamoxifen 239. Una mujer de 62 años con un carcinoma ductal infiltrante T2 N0 M0 receptores estrogénicos positivos le

pregunta con respecto al Tx con Tamoxifen. Usted le aconseja que: (verificar puede ser opción c) a. El Tamoxifen no está indicado en mujeres post-menopáusicas. b. Debe tomar Tamoxifen por cinco años. c. Debe tomar Tamoxifen por tres años d. Si los ganglios son negativos no necesita tomar Tamoxifen. e. Antes de tomar una decisión se debe esperar la determinación del marcador Her-2 neu 240. Mujer de 60 años acude por presentar una masa en CSE de la mama derecha, tiene mamografías que es

reportada como UNAM asa de 6 cms sin signos mamográficos de malignidad. El próximo paso a seguir sería: a. Biopsia b. Nueva mamografía en 3 meses c. Mastectomía d. AINES y re-evaluación en 15 días e. Biopsia solo si tiene factores de riesgo.

241. Una paciente de 45 años se le ha diagnosticado un carcinoma ductal infiltrante T3 N1 M0 el próximo paso

a seguir sería.

a- Mastectomía radical modificada b- Quimioterapia neoadyuvante c- Radioterapia d- Seguimiento en 1 mes e- Tamoxifen

Page 46: Gran Compendio Final Cirugia

242. Una Dama de 45 años, le consulta por presentar un hallazgo mamográfico caracterizado por microcalcificaciones en acúmulo, con sospecha de tumoración. Categorizada como BIRADS IV. Al examen físico no hay tumoración palpable. El próximo paso a seguir sería:

a. Biopsia b. Estudios genéticos c. Observación d. Resonancia magnética de la mama e. Ultrasonido mamario 243. A una mujer de 58 años se le detecta un tumor de 2 cms que es resecado y reportado como carcinoma

ductal infiltrante con márgenes libres. El próximo paso a seguir sería:

a- Biopsia de ganglio centinela b- Mastectomía radical modificada c- Quimioterapia neo-adyuvante d- Radioterapia e- Tamoxifen

244. Las ventajas de la Mastectomía radical modificada sobre la terapia con cirugía conservadora incluyen.

a- Mayor sobre vida b- Menores recurrencias locales c- Menor incidencia de metástasis a distancia d- Menor morbilidad e- Ninguna de las anteriores.

245. Una mujer de 55 años tiene una tumoración en CSE de la mama derecha de 4 cms de diámetro, es nodular

cauchosa, indolora y bien definida, tiene 6 meses de evolución. Usted le recomienda: a- Mamografía y biopsia b- Mamografía y biopsia solo si sospecha malignidad c- No hacer nada, es un fibroadenoma d- Mamografía ahora y repetirla en tres meses para ver si hay cambios. e- Re-evaluar en dos semanas.

Page 47: Gran Compendio Final Cirugia

246. Un Hombre alcohólico, de 65 años, consulta por un área indurada de reciente aparición en la mama izquierda. Al examen físico usted diagnostica ginecomastia izquierda con una masa dominante en el C S E y una adenopatía endurecida en la axila ipsilateral. El próximo paso a seguir será:

a. Biopsia b. Estudios genéticos c. Evaluación psiquiátrica para que de deje de tomar d. Resonancia magnética de la mama e. Ultrasonido mamario

247. Pueden ser causas de ginecomastia todas excepto:

a- Marihuana b- Cimetidina c- Cirrosis hepática d- Ibuprofeno

248. Todas las mujeres deben realizarse mamografías de pesquiza anuales:

a- A partir de los 20 años b- A partir de los 30 años c- A partir de los 40 años d- A partir de los 50 años

249. Una muchacha de 16 años, consulta por una tumoración cauchosa móvil, lobulada de más o menos 3 cms.

de diámetro en el CSE de la mama izquierda, de más o menos seis (6) meses de evolución. Todos los siguientes serán apropiados en el manejo de esta Paciente excepto: a. Biopsia b. Punción con aguja fina c. Reevaluación en tres (3) meses. d. Resonancia magnética de la mama e. Ultrasonido mamario. 250. Una adolescente de 16 años tiene una tumoración bien definida de 5 cm. de diámetro, en el cuadrante

infero-interno de la mama D, es móvil, lobulado, cauchosa e indolora. El tratamiento debe consistir en: a. Ultrasonido y excisión del fibroadenoma. b. Mamografía. c. Incisión y drenaje de absceso. d. AINES.

Page 48: Gran Compendio Final Cirugia

251. Al evaluar una masa mamaria son signos de mal pronostico todos excepto: a. Edad mayor de 40 años. b. Biopsia previa que evidenció cambios proliferativos. c. Cáncer de mama en una hermana. d. Secreción verdosa a través del pezón. 252. Según la Sociedad Americana de Cáncer las guías a seguir para el tamizaje del cáncer de mama son

excepto: a- Toda paciente mayor de 40 años se debe realizar mamografía anual. b- Pacientes menores de 20 años se deben hacer auto examen de la mama mensualmente. c- Pacientes menores de 38 años deben ser examinadas las mamas anualmente por un médico. d- Pacientes de 30 años con incremento del riesgo de cáncer de mama se debe realizar cada 6 meses mamografía y ultrasonido de mama.

253. Todo es cierto acerca de la mamografía excepto:

a- Es útil en la mujer asintomática. b- El 10-15% de los canceres de mama no se observan en la mamografía. c- La mayoría de los canceres de mama no se palpan. d- Es útil para biopsias dirigidas.

254. En una mujer de 23 años todo los siguientes son ciertos excepto:

a- Debe efectuarse un auto examen manual mensual de las mamas b- Sus mamas deben ser examinadas por un médico por lo menos una vez al año. c- La mamografía es de poca utilidad porque hay que irradiar mucho a las pacientes. d- El auto examen manual se debe realizar después de la menstruación

255. Paciente de 25 años de edad consulta por presentar fiebre, aumento de volumen de la mama derecha y dolor. Refiere que esta amamantando a su niña que tiene 1 año. Al examen la mama se encuentra roja y dura. Usted debe:

a- Enviar al SOP para incisión y drenaje. b- Mamografía. c- Antibióticos, calor local y AINES. d-Tomar biopsia.

256. Pte de 60 años con cancer intraductal mamario, tumor de 2 cm. Sin ganglios palpables en axila, el mejor

tratamiento a seguir es:

a- Mastectomía radical modificada. b- Lumpectomía más vaciamiento axilar sin radio Tx post-op. c- Lumpectomía más biopsia del ganglio centinela con radio Tx post.op d- Lumpectomía solamente. e- Ninguna de las anteriores

257. Son signos de mal pronóstico en el cáncer de mama. Todos excepto: a- Tumor mayor de 4 cms b- Ganglios positivos en axila c- Carcinoma lobular in situ d- Receptores estrogénicos negativos

Page 49: Gran Compendio Final Cirugia

e- No hay exceptos, todos son signos de mal pronóstico.

258. Una paciente tiene un tumor de 6 cms en la mama. Una biopsia con aguja diagnostica carcinoma ductal infiltrante. El mejor Tx inicial a seguir sería:

a- Quimio terapia pre-operatoria. b- Mastectomía radical modificada. c- Lumpectomía más vaciamiento axilar con radio terapia post-operatoria. d- Lumpectomía más vaciamiento axilar sin radio terapia post.op. e- Ninguna de las anteriores

259. Para cada una de las siguientes preguntas escoja una de las siguientes opciones

a) Carcinoma ductal in situ b) Carcinoma lobular in situ c) Ambos d) Ninguno

B Más común en mujeres premenopáusicas B Nunca es palpable A Hallazgos mamográficos significativos C Riesgo de cáncer invasivo mas o menos 30% B Igual riesgo de cáncer invasivos en ambas mamas A Cáncer invasivo se presenta en el mismo sitio del cáncer in situ 260. Para cada una de las siguientes preguntas escoja una de las siguientes opciones a. Carcinoma lobular in situ b. Carcinoma ductual in situ c. Ambos d. Ninguno B rara vez es palpable A Nunca es palpable A Es un factor de riesgo B Puede avanzar a carcinoma invasor B Lumpectomía sin vaciamiento axilar A Observación A Mastectomía profiláctica D Quimioterapia pre-operatoria B Quimioterapia post-operatoria B Microcalcificaciones Balance Hidroelectrolítico 261. Usted está en su primer día de internado y el residente le pide que le corrija la hiponatremia a un

paciente. Usted debe tomar en cuenta lo siguiente: a. Hacer los cálculos del déficit y suministrar la mitad en 8 horas

Page 50: Gran Compendio Final Cirugia

b. La reposición no debe ser mayor d 20 mEq/h c. Examinar al paciente y revisar el expediente clínico d. Darle diurético al paciente

262. Usted está en cuidados intensivos de cirugía y es su primer día de internado. La enfermera lo llama

porque el paciente operado de bypass coronario no orina, luego de haber leído el expediente, que debe hacer usted:

a. Aumentar los líquidos intravenosos b. Darle diurético al paciente c. Decirle a la enfermera que lo evalúe en una hora más d. Revisar la sonda foley en otros dice colocar

263. Usted está en sala 9 y es su primer día de internado. La enfermera lo llama porque el paciente operado de

colecistectomía no orina, luego de haber leído el expediente, que debe hacer usted: a. Aumentar los líquidos intravenosos b. Darle diurético al paciente c. Decirle a la enfermera que lo evalúe en una hora más d. Revisar la sonda foley

264. Causa de brecha aniónica elevada: a. Hipoperfusión b. Diarrea c. Fístula pancreática d. Excesiva administración de ácidos

265. La cantidad de mEq/L de sodio que contiene un L lactato ringer es de a. 130

b. 154 b. 200 c. 145 d. Ninguna de las anteriores

266. Paciente de 80 años hospitalizado en la unidad de cuidados intensivos revela los siguientes gases

arteriales ph 7.25 PCO2 92 mmHg bicarbonato 14 mEq/L El trastorno ácido base que esta padeciendo es: a. Acidosis respiratoria b. Alcalosis respiratoria c. Acidosis metabólica d. Alcalosis metabólica

243. Paciente masculino de 30 años de edad que recibe múltiples impacto de bala en la región abdominal es llevado al salón de operaciones realizándole laparotomía exploradora encontrando devascularización del intestino delgado. Se le realizó resección de más o menos 2 metros de intestino delgado y anastomosis. La cirugía tuvo una duración de 4 horas y se le transfundieron 4 unidades de GRE. En el postoperatorio se encuentra muy distendido con un drenaje aumentando a través del tubo de naso gástrico Tiene un peso de 80 Kg. Sus electrolitos son: Na 125 meq, K 3,0 meq, Cl 80 meq. Creatinina 2,5. Bun 48. Glc 180. Gases arteriales 7,4. HCO3 30 meq, PCO2 56 mm Hg. PO2 96 mm Hg y Sat de oxígeno 100 %. Exceso de Base más 4.

Page 51: Gran Compendio Final Cirugia

267. La diuresis horaria de este paciente no debe ser menor de: (verificar a,c) a. 30 cc por hora b. 50 cc por hora c. 40 cc por hora d. 10cc por hora

268. La causa de la hiponatremia es de tipo: a. Hipotónica – Hipovolemica b. Hipotónica - Isovolémica. c. Hipotónica - Hipervplemica. d. Isotónica.

269. La osmolaridad calculada de este paciente es: a. 200 mosm b. 280 mosm c. 150 mosm d. 260 mosm

270. El déficit de sodio calculado es: a. 600 meq. b. 560meq. c. 720 meq d. 920 meq 271. El desequilibrio ácido – base que presenta es: a. Alcalosis metabólica compensada. b. Alcalosis metabólica descompensada.

272. El déficit de potasio de este paciente es: a. Menos de 100 meq b. 150 meq c. 250 meq d. Mayor de 200 meq.

273. En sala tiene un paciente, el cual diagnostica una hiponatremia y el déficit de sodio calculado es de 200

meq . los líquidos que usted debe ordenar es: a. SSN 500 cc + NaCl a 23,4% 4 cc. Pp en 10 hrs. b. L/R 1000 cc + NaCl a 23,4% 18 cc. Pp en 10 hrs. c. SSN 1000 cc + NaCl a 23,4% 60 cc pp en 10 hrs. d. SSN 1000 cc + NaCl a 23,4% 10 cc p p en 10 hrs.

Shock

Page 52: Gran Compendio Final Cirugia

274. En cuanto a la relación a la patogénesis del shock séptico y sepsis podemos decir que: a. El SIRS se puede producir solo en pacientes de trauma b. La sepsis se define como un paciente que presenta SIRS, foco infeccioso e hipotensión irreversible c. La isquemia intestinal producida en el shock promueve la aparición de MODS por aumento de la

translocación bacteriana d. La nutrición enteral y parenteral no están indicados en pacientes con sepsis por el riesgo de resistencia

periférica a la insulina 275. El signo más temprano de hipovolemia en un paciente es: a. Hipotensión b. Taquicardia c. Mucosa seca d. Disminución de la diuresis e. Estado de conciencia 276. En cuanto a la relación del intestino en la patogénesis del shock

séptico y sepsis podemos decir que: a. La recontaminación intestinal selectiva con antibióticos orales a disminuido las infecciones y la mortalidad b. La utilización de fármacos anti-inflamatorios ha logrado modular la respuesta inflamatoria en el síndrome de

disfunción multiorgánica c. La isquemia intestinal producida en el shock promueve la aparición de MODS por aumento de la

translocación bacteriana d. La nutrición parenteral preserva las vellosidades intestinales 277. Según American C.. y Care Medicine 1991, se define SIRS como: a. Signos inespecíficos como temperatura > 38C o < 36, leucocitosis o leucopenia, taquipnea y taquicardia b. Signos inespecíficos como fiebre, leucocitosis, taquicardia, taquipnea y gérmenes detectados en hemocultivo c. Situación de Shock asociada a signos clínicos inespecíficos como temperatura> 38C o <36C, leucocitosis o

leucopenia, taquipnea y taquicardia d. Infección diagnosticada microbiologicamente acompañada de hipotensión y oliguria e. Hipotensión <90 mmHg mantenida a pesar de perfusión de drogas inotrópicas 278. Según American C.. y Care Medicine 1991, se define SEPSIS como: a. Signos inespecíficos como temperatura > 38C o < 36, leucocitosis o leucopenia, taquipnea y taquicardia b. Signos inespecíficos como fiebre, leucocitosis, taquicardia, taquipnea y gérmenes detectados en

hemocultivo c. Situación de Shock asociada a signos clínicos inespecíficos como temperatura> 38C o <36C, leucocitosis o

leucopenia, taquipnea y taquicardia d. Infección diagnosticada microbiologicamente acompañada de hipotensión y oliguria e. Hipotensión <90 mmHg mantenida a pesar de perfusión de drogas inotrópicas 279. Paciente de 35 años el cual es admitido al SOP por haber sido atropellado por un autobús con múltiples

lesiones abdominales. Transoperatoriamente se trato agresivamente el shock hipovolémico y la cirugía tuvo una duración prolongada aproximadamente de 6 horas. Luego de 48 horas usted está en la unidad de cuidados intensivos de turno y la enfermera le dice que no está orinando a pesar que tiene buena presión y la PVC es normal. Usted luego de revisar la sonda urinaria decide colocar un diurético pero no presenta diuresis.

a. Este paciente probablemente tiene una trombosis de la arteria renal

Page 53: Gran Compendio Final Cirugia

b. Posiblemente su presión abdominal es mayor de 15 mmHg c. Se debe iniciar diálisis renal d. Aumenta el goteo de los líquidos intravenosos

280. El tx inmediato del paciente anterior debe ser: a. Heparina y estreptoquinasa b. Medir la presión intraabdominal c. Realización de fístula vascular para hemodiálisis d. Colasión de filtro en la vena cava

281. Los líquidos utilizados en la resucitación del paciente en un shock

son los siguientes, excepto: a. Cristaloides b. Coloides c. Hemoderivados d. Dextrosa al 5%

Page 54: Gran Compendio Final Cirugia

282. Están aumentados en el shock todos, excepto: a. Vasopresina b. Epinefrina c. pCo2 d. Cortisol e. Acido Láctico

Heridas y Suturas 283. Para el tx con antibióticos de las heridas se debe tomar en cuenta lo siguiente: a. Tipo de herida b. Factores del paciente c. Mecanismo de lesión d. Todas las anteriores 284. Con respecto a la preparación pre-operatoria del paciente: a. El rasurado debe ser extenso y meticuloso b. El rasurado se debe realizar la noche anterior a la cirugía c. La preparación con yodo es bactericida, fungicida y virucida d. La tintura de yodo es un iodoforo altamente efectivo 285. En cuanto a las suturas quirúrgicas todas son ciertas, excepto: a. Las suturas absorvibles producen menos reacción tisular que las no reabsorvibles b. Las suturas polifilamentosas tienden a favorecer las infecciones c. Una sutura 3-0 es más delgada que una 2-0 d. El ácido poliglicólico es una sutura que se absorve por hidrólisis

286. Diga 2 ejemplos de herida según clasificación por el riego de infección: a. Limpia: b. Limpia Contaminada: c. Contaminada: d. Sucia:

287. En cuanto a los antisépticos y el tx de las heridas todo es cierto,

excepto: a. Los antisépticos retrasan el proceso de cicatrización b. No se deben utilizar en heridas limpias c. Su penetración es superficial d. El peróxido de hidrógeno es un potente bactericida

288. Con respecto a los antibióticos profilacticos todo es cierto,

excepto: a. Están indicados en colecistotomias sin factores de riesgo b. Están indicados en apendicectomias c. Están indicados en cirugías por obstrucción pilórica d. Deben ser administrados justo antes de iniciar la operación

Page 55: Gran Compendio Final Cirugia

Neoplasias Gástricas 289. Todos los siguientes son signos de neoplasia gástrica avanzada: a. Nódulo de Wichow b. Nódulo de sister mary Joseph c. Signo de Blumer d. Todas las anteriores e. Solo a y c

290. Factor etiológico de mayor importancia en el desarrollo del Cáncer Gástrico es:

a- Bajo consumo de vegetales. b- Alto consumo de almidones. c- Alto consumo de alimentos salados y ahumados d- Infestación por el Helicobacter pylori.

e- Ninguna de las anteriores.

291. Se debe sospechar de Gastrinoma o Síndrome de Zollinger-Ellison en el siguiente caso: a- Ulceras resistentes a tratamiento antisecretorio. b- Múltiples ulceras duodenales. c- Ulceras que recurren luego de procedimiento quirúrgicos adecuados. d- Todas las anteriores. e- Ninguna de las anteriores.

292. según la clasificaciòn de Ca Gastrico de Borman la linitis plàstica es el tipo: a. tipo I b. tipo II c. tipo III d. tipo IV e.tipo V

Page 56: Gran Compendio Final Cirugia

293. todos los siguientes son signos de neoplasia gàstrica avanzada, excepto:

a. nodulo de vircow b. nodulo de sister mary joseph c. signo de blummer d. signo del Moro e.ninguna de las anteriores

294. Pte de 75 años el cual es traído al cuarto de urgencias del HST luego de haber sido tratado en el centro de

Salud con antiácidos y metoclopramida con el diagnostico clínico de gastritis. Los familiares informan que el señor ha perdido peso y que desde hace un año lo estaban llevando al centro de salud porque se quejaba de dolor en epigastrio además de presentar vómitos post-prandriales. También han notado la aparición de una masa sobre la clavícula izquierda. El estudio de elección de este paciente inicialmente debe ser:

a. Tomografía axial computarizada. b. Endoscopia digestiva alta. c. Colonoscopia.

d. Biopsia de la masa clavicular izquierda. 295. Hombre de 68 años se presenta a su consultorio quejándose de disfagia progresiva durante los últimos

tres meses, relacionada con incomodidad moderada en el pecho , el paciente refiere una pérdida de peso de 7,5 Kg. , tiene antecedente de tabaquismo positivo durante 30 años, e ingesta de alcohol ocasional. A la exploración física sin datos de interés, incluso los signos vitales, una radiografía de tórax es normal y el esofagograma con bario muestra un defecto de llenado irregular en el tercio distal del esófago con deformación y estrechamiento de la luz:

¿Cuál de los siguientes diagnósticos es el más probable? a- Esofagitis por estenosis. b- Carcinoma de esófago. c- Carcinoma pulmonar con invasión a esófago. d- Divertículo de Zenker. 296. El carcinoma típico que se desarrolla en asociación con el esófago de Barrett es:

a- Epidermiode. b- Mucoepidermoide. c- Células pequeñas. d- Adenocarcinoma. e- Células escamosas.

297. ¿Cual es la afirmación correcta acerca del cáncer gástrico?

a- El cáncer gástrico distal es el más común actualmente. b- Los cánceres gástricos tempranos están confinados a la mucosa y submucosa. c- Los tumores gástricos tipos difusos tiene mejor pronósticos que el tipo intestinal.

d- Todas las anteriores son ciertas. 298. su tia de 60 años que desde hace 4 semanaas se vanìa sintiendo con una sensaciòn de llenura post-

prandial y dolor ocasional en FID. Usted le recomienda que: a. se haga mamografìa b. se haca una colonoscopia c. se haga un Pap cervicouterino d. todas las anteriores.

Page 57: Gran Compendio Final Cirugia

299. Son todas características de la motilina, excepto: a. Es un péptido intestinal b. Es secretado por las células enterocromafines del intestino delgado c. La acidficación del duodeno causa aumento de su liberación d. La eritromicina inhibe su liberación

Enfermedades del Hígado 300. Lesión benigna más común encontrada en el hígado: a. Metástasis neoplásicas b. Hepatocarcinoma c. Hemangioma d. Abcesos hepáticos 301. La ecuación de Harris Benedict nos sirve para: a. Cuantificar las proteínas totales b. Cuantificar los carbohidratos c. Nos proporciona el gasto energético basal d. Cuantificar los lípidos e. Todas las anteriores

Page 58: Gran Compendio Final Cirugia

302. La irrigación del hígado esta dada por: a. Arteria hepática 25%, vena porta 85% b. Arteria hepática 75%, vena porta 25% c. Arteria hepática 25%, vena porta 75% d. Arteria hepática 50%, vena porta 50% Enfermedades del Bazo 303. Primer signo que sugiere rotura traumática del bazo: a. Inconciencia b. Dolor abdominal c. Hipotensión, palidez d. Hematemesis, taquicardia, hipotensión

ERGE 304. El esófago inferior de un paciente es inadvertidamente perforado durante una endoscopia. Un trago de

bario hecho inmediatamente luego de la perforación demuestra salida de bario hacia el espacio pleural izquierdo. Cuál es el tx adecuado más aceptado:

a. Observación b. Observación más antibióticos c. Toracotomía cerrada izquierda d. Intubación esofágica prolongada e. Drenaje y reparación quirúrgica de la lesión

Anestésicos 305. Una paciente de 70 kg que llega al cuarto de urgencias con una herida de cuello amplia y que usted va a

suturar, se debe considerar: a. La dosis máxima de xilocaína que debe infiltrarse es 350 mg b. La dosis máxima de xilocaína que debe infiltrarse es de 250 mg c. La xilocaína viene en concentración de 50 mg/cc d. La vida media de la xilocaína es aprox 4 horas 306. Cuantos mg de lidocaína contienen 30 cc de lidocaína al 2%: a. 200 mg b. 300 mg c. 400 mg d. 500 mg e. 600 mg

Page 59: Gran Compendio Final Cirugia

307. La clasificación de ASA nos orienta sobre: a. El riesgo de mortalidad operatorio b. El riesgo de complicaciones peri operatorias c. El riesgo cardiológico peri operatorio del paciente d. Ninguna de las anteriores

308. La ventaja de la analgesia controlada por el paciente es: a. El paciente tiene control sobre su medicamento b. Independencia de la enfermera c. Inmediata medicación d. Rápida analgesia e. Todas las anteriores

309. Durante la etapa pre anestésica podemos decir que: a. La clasificación de ASA correlaciona el estado general del paciente y el riesgo anestésico b. La clasificación de Mallmpati correlaciona el estado general del paciente y el riesgo anestésico c. Los pacientes ASA V, son siempre de intubación difícil d. Los pacientes Mallampati gradi I, son siempre de intubación dificil e. Los pacientes ASA I corrientemente presentan problemas durante el acto anestésico

310. El orden de aparición clínica de signos de intoxicación por anetésicos locales es: a. Adormecimiento de la lengua, confusión, paro cardíaco, paro respiratorio b. Adormecimiento de la lengua, confusión, paro respiratorio, paro cardiaco c. Confusión, adormecimiento de la lengua, paro respiratorio, paro cardiaco d. Confusión, adormecimiento, paro cardiaco, para respiratorio Insuficiencia… (circulación) 311. La triada de Virchow consta de: a. Estasis, edema y hipercoaguabilidad b. Estasis, enrojecimeinto cutáneo y edema c. Estasis, hipercoaguabilidad, lesión vascular d. Ninguna de las anteriores

312. Paciente con riesgo aumentado de trombosis venosa elevado, excepto: a. Encamados b. Uso de anticonceptivos orales c. DM d. Insuficiencia cardiaca

313. Todo es cierto de los aneurismas abdominales, excepto: a. Los aneurismas mayores de 5cm tienen mayor riesgo de ruptura b. Su etiología es multifactorial c. Es una enfermedad de ancianos d. La mayoría de los aneurismas son asintomáticos

Page 60: Gran Compendio Final Cirugia

314. El factor de mal pronóstico más importante en el shock hemorrágico por traumatismo es: a. El aporte masivo de volumen en la atención pre hospitalaria b. El tiempo transcurrido desde el impacto hasta la cirugía c. La atención realizada en el hospital de referencia d. La realización de las pruebas dx apropiadas e. Existencia de lesiones ortopédicas

315. Usted tiene una paciente que tiene dx de TEP, se le inicia la Heparinización con un bolo de 80 unidades y una infusión de 18 unidades por hora. Se toma un control de TPT a las 4 horas y es el siguiente TPT P35 TPTC 35, usted debe: a. Bolo de 80 unidades de heparina b. Bolo de 40 unidades de heparina c. Disminuir la infusión y esperar 2 horas d. Bolo de 100 u de heparina

316. Subitamente el paciente anterior, luego de haberle administrado 5000 u de heparina comienza a

presentar hematemesis y por los sitios de punción hay sangrado espontáneo, se realiza un control de TPT y es informado así, TPTP es >1 minuto. Usted debe:

a. Detener la heparina b. Detener la heparina + 50 mg de sulfato de protamina c. Detener heparina + 4 u de plasma fresco congelado + 50 mg de sulfato de protamina d. Detener la heparina y trnsfundir 3 u de plasma congelado

Page 61: Gran Compendio Final Cirugia

317. Todo los siguientes son síntomas de la insuficiencia arterial, excepto: a. Atrofia muscular b. Edema de miembros inferiores c. Vello escaso d. Rubor Obesidad Morbida 318. El índice de masa corporal de un paciente que pesa 180 kg y mide

1.70 metros es de: a. 105 b. 62 c. 181 d. 0.009 e. Ninguna de las anteriores Patología Colónica 319. Son indicaciones para colocación de drenajes, excepto: a. Luego de una sutura hepática secundaria a un traumatismo b. Luego de una coledocotomia para explorar la vía biliar c. Luego de una mastectomía radical modificada d. Luego de una laparotomía exploradora por peritonitis generalizada sin encontrar el foco infeccioso

320. Mujer de 70 años cardiopata acude por dolor abdominal, distensión abdominal y obstinación, la rx

muestra distensión del colon, con poco gas en intestino delgado, el ciego mide 11 cm no se ve gas en el recto, todas son cierta, excepto:

a. Tiene una obstrucción en asa ciega b. La causa más probable de su problema es una neoplasia del sigmoides c. La perforación colónica es inminente d. Necesita una colonoscopia urgente

321. Entre los cuidados de los drenajes se debe considerar: a. Se deben cambiar periódicamente los apósitos b. Se debe calcular cuanto es lo drenado periódicamente c. Se debe reponer en los líquidos administrados al paciente lo drenado d. Todas las anteriores e. Solo b y c

Page 62: Gran Compendio Final Cirugia

322. Un hombre de 35 años acude por dolor perianal que no le permite sentarse, fiebre y diarrea de 4 días de evolución, al examen físico la región perianal es normal, excepto por un endurecimiento doloroso del lado derecho que hace muy difícil el examen, el manejo debe ser:

a. Baños de asiento por hemorroide izqueirda b. Antibiótico por su gastroenteritis c. Incisión y drenaje del abceso perianal d. Laxantes por su fisura anal e. Ninguna de las anteriores

323. Paciente de 80 años operado por enfermedad diverticular

perforada del colon, se le realiza hemicolectomía izquierda y colostomia terminal. A los 5 días presenta febrícula, aumento de necesidad de líquidos para mantener diuresis, además hiperglicemias y distensión abdominal. Sería recomendable en este paciente efectuar:

a. Hemocultivo b. USG abdominal c. CAT de abdomen d. Centelleo con leucocitos marcado con tecnecio 99

Pie Diabético 324. La forma más efectiva de prevenir la aparición de ulceras en los pies de los pacientes diabéticos es: (C o

D) a. Arteriografía temprana. b. Medición de la oximetría del pie c. Detección temprana de pérdida de la sensibilidad. d. Utilización de calzados especiales.

325. Paciente de 50 años de edad diabético tratado con hipoglucemiantes orales, quien desde hace una

semana presenta fiebre y aumento de volumen del pie derecho. Refiere que hace un mes presentaba ulcera plantar en pie derecho. Todas las siguientes son causas intrínsecas de ulceración excepto: (a o d)

a. Neuropatía. b. Ceguera c. Pobre conocimiento de la diabetes. d. Ulcera previa.

Perforación Intestinal

Page 63: Gran Compendio Final Cirugia

326. La causa más común de perforación intestinal en pacientes con SIDA es: a. Linfoma b. Sarcoma de Kaposi. c. Gastroenteritis x CMV d. Infecciones por Mycobacterias. Enfermedad De Hirchprung

327. Todo lo mencionado para la Enfermedad de Hirchsprung es cierto, excepto: a- Se caracteriza por ausencia de células ganglionares. b- El segmento más afectado en el colon ascendente e íleon terminal. c- El enema baritado y la biopsia rectal son fundamentales para el diagnóstico. d- Sospechamos diagnóstico en el neonato que no expulsa meconio en las primeras 24 horas de vida.

Respuesta Inflamatoria Sistémica 328. El síndrome de respuesta inflamatoria sistémica se define como dos o mas de los siguientes enunciados

excepto: a. Temperatura mayor de 38º. b. Frecuencia cardiaca mayor de 90 x c. Frecuencia respiratoria mayor de 20 x d. Leucocitos mayor de 12 000. e. % de bandas mayor de 4%.

329. Es causa de síndrome de respuesta inflamatoria sistémica: f. Pancreatitis aguda. g. Quemaduras de segundo grado 30% de SCT. h. Infección por CMV. i. Herida por arma de fuego en abdomen. j. Todas las anteriores.

330. la citoquina que se cree que es la màs importante mediadora del SRIS SDMO es:

1. TNF 2. IL-1 3. IL-6 4. IL-8 5. Ninguna de las anteriores

331. El tratamiento de SDMO para mejorar la cinètica del oxigeno dee consistir en: a. mojorar oxigenacion b. optimizar la relacion DO2/VO2 c. Corregir la anemia d. Debridamiento de tejido necròtico e. Control de hipertermia f. Todas las anteriores g. Ninguna de las anteriores

Pareo.

a. Lynch _____ Fiebre, dolor con ictericia

Page 64: Gran Compendio Final Cirugia

b. Ogilvie _____ Fiebre, dolor, ictericia, obnulación e hipotensión.

c. Klatskin _____ Tumor de la confluencia de los conductos hepáticos. d. Reynolds _____ Pseudo obstrucción colónica e. Charcot _____ Cancer colorectal hereditario no polipósico

332. Diga 5 características del dolor que siempre deben ser constatadas como parte del interrogatorio ante un

cuadro de dolor abdominal .

a. Inicio b. Migracion c. Tipo d. Que lo mejora e. Que lo empeora CICATRIZACIÓN 333. Todo lo siguiente es cierto acerca de la cicatrización de las heridas quirúrgicas excepto:

a- Luego de 24 horas no es necesario cubrir la herida. b- En la fase de inflamación hay migración de leucocitos, monocitos y macrófagos hacia la herida. c- El cierre de la herida por segunda intención se sutura luego de 5 días cuando no hay signos de

infección. d- La fase de contracción solo ocurre en heridas grandes.

Page 65: Gran Compendio Final Cirugia

334. La cicatrización por segunda intención es: a. cuando el tejido es icidido y suturado b. cuando el tejido es incidido, debridado y suturado. c. cuando ocurre en heridas abiertas y hay formación de tejido de granulacion con recubrimiento final de la

herida. d. Cuando la herida cure abierta durante unos 5 dias y despues se cierra. e. Ninguna de las anteriores

335. la respues de cicatrizacion se inicia: a. al momento de la lesion b. cuando la sangre es expuesta a la colágena c. cuando se inician las curaciones d. a las 24 horas después de la lesión

336. los principales componentes celulares de la cicatrización son : a. granulocitos, linfocitos y plaquetas. b. Linfaticos elementos trazas y enzimas c. Macrófagos y fibroblasto d. Todas las anteriores. e. Solo a y c VESICULA Y VIAS BILIARES / ICTERICIA 337. El objetivo inicial de la terapia de la colangitis aguda es: a. Aliviar la ictericia y prevenir el daño hepático b. Prevenir el desarrollo de la pancreatitis c. Remover el cálculo si está presente d. Descomprimir la vía biliar

338. Se le realiza a una paciente colecistectomía y exploración de vías biliares, 2 cálculos fueron removidos y

se colocó un tubo en T en el colédoco, a los 9 días post operatorios, se le elevan los niveles de bilirrubina y tiene dolor en hipocondrioderecho irradiado a su espalda. Cuál debe ser el próximo paso en el manejo de esta paciente:

a. Iniciar terapia con drogas que disuelvan los cálculos b. Remover el tubo T c. Pinzar el tubo en T d. Operar a la paciente e. Realizar colangiografía por tubo en T

Page 66: Gran Compendio Final Cirugia

339. El dolor típico de la colelitiasis es a. En epigastrio o hipocondrio derecho, cólico, menos de una hora de duración. b. En epigastrio o hipocondrio derecho, constante, minutos de duración. c. En epigastrio o hipocondrio derecho, cólico, minutos de duración. d. En epigastrio o hipocondrio derecho, constante más de una hora de duración. e. En epigastrio y espalda, constante, menos de una hora de duración. 340. Paciente de 65 años de edad se presenta por presentar molestias abdominales e ictericia durante las

últimas tres semanas. Tiene antecedentes de tabaquismo positivo por 30 años, e ingesta de alcohol ocasional, y un melanoma ulcerado de 5,5mm que fue retirado de su espalda hace dos años y medio. La exploración física mostró un paciente ligeramente ictérico, con los signos vitales normales y el abdomen ligeramente distendido con dolor en el cuadrante superior derecho y hepatomegalia importante. El diagnostico más probable es:

a. Hepatitis A. b. Metástasis hepáticas. c. Pancreatitis. d. Hepatoma. 341. La evaluación del paciente anterior debe incluir:

a. Anti A IgM. b. Amilasa y lipasa serica. c. Tomografía axial computarizada. d. Alfa feto proteína.

342. Para cada una de las siguientes preguntas escoja una de las siguientes opciones:

a. Pancreatitis aguda b. Pancreatitis crónica c. Ambas d. Ninguna

D Shock C Dolor abdominal B Diabetes y esteatorrea B Es necesaria cirugía en más del 70% de los casos C Pseudoquistes

343. Una paciente de 80 años acude al Servicio de Urgencias con fiebre de 39°C, Ictericia, G.B 20,000 con 85%

neutrófilos. El manejo inicial de esta paciente debe incluir todos excepto. a. Antibióticos b. Laparoscopia c. Perfil de coagulación d. Reanimación cardiovascular e. Ultrasonido

344. Una mujer de 78 años consulta por fiebre con escalofríos, dolor abdominal e ictericia. Usted diagnostica

colangitis aguda.  La PA es de 80/70 FC IIS x’.  El tratamiento a seguir es.

Page 67: Gran Compendio Final Cirugia

a. Antibiótico terapia por 48 hrs. b. Drenaje biliar urgente c. Catéter de Swan-Ganz para establecer la causa del shock. d. Tomografía computarizada e. Tratamiento en Unidad de Cuidados Intensivos.

Para cada una de las siguientes preguntas escoja una de las siguientes opciones a. ERCP b. Colangio Resonancia c. TAC d. Ninguno 345. _B_ Bilirrubinas totales 8 mg % F. Alcalina 100 (N < 125) a. US: vías biliares 9 mm (N < 6 mm), sin coledocolitiasis 346. _A_ Bilirrubinas totales 8 mg% F. Alcalina 200 (N < 125) a. US: vías biliares 9 mm muestra coledocolitiasis 347. _C_ Bilirrubinas totales 15 mg.% F. alcalina 500 (N < 125) a. US: vías biliares 12 mm masa en cabeza de páncreas y colelitiasis 348. _D_ Bilirrubinas totales 10 mg % F. alcalina 100 (N < 125) a. US: vías biliares 5 mm ( N < 6 mm) sin colelitaisis

Page 68: Gran Compendio Final Cirugia

349. Con respecto a la colelitiasis todas son ciertas excepto: a. 60% de pacientes con colelitiasis nunca tendrán síntomas b. La dispepsia y mareos son indicaciones de cirugía en pacientes con colelitiasis c. 2% de los pacientes con colelitiasis tendrán como primer síntoma una complicación d. Ninguna de las anteriores. 350. Con respecto a la coledocolitiasis todas son ciertas excepto: a. Su incidencia es el 10-15% en pacientes con colelitiasis b. La mayor parte de los cálculos son primarios del propio colédoco c. La prueba funcional hepática mas sensible es la fosfatosa alcalina d. Es una causa frecuente de colangitis 351. Los pacientes con colangitis pueden presentar todos los siguientes excepto: a. Fiebre con escalofríos b. Ictericia c. Dolor en cuadrante superior derecho d. Obstrucción intestinal 352. Una paciente de 80 años tiene un cuadro de colangitis aguda de 36 horas de evolución por

coledocolitiasis. Un intento de descomprimir la vía biliar por CPRE ha fallado. El radiólogo intervencionista está fuera de la ciudad. Su PA es 80/40, Fc 120x'. Recibe antibióticos apropiados desde hace 36 horas. El próximo paso a seguir será.

a. Cirugía b. Cambio de antibióticos c. Otro intento de CPRE dentro de 24 horas d. Esperar que regrese el radiólogo intervencionista en 24 horas

353. Describa un " Cólico biliar " típico a. Dolor de Ubicación en: HCD o epigastrico b. Constante o Espasmódico( escriba cual): CONSTANTE c. Duración en horas mas de UNA y menos de 24HORAS d. Las náuseas son frecuentes si ó no: SI e. Siempre está asociado a la ingesta de alimentos si ó no: PUEDE O NO ESTAR ASOCIADO A LA INGESTA DE

ALIMENTOS 354. La vena porta tiene su origen de la confluencia de:

a. La vena esplènica y la vena mesentérica inferior b. La vena mesentérica superior y la vena esplénica c. La vena cava inferior y la vena esplénica d. Ninguna de las anteriores

355. Un hombre de 60 años, diabético consulta por historia de mas o menos 24 horas de evolución

caracterizada por dolor en HCD, fiebre de 38.5C, G.B. 19.0 con 95% N. El US es compatible con colecistitis aguda calculosa. Todas las siguientes son ciertas excepto:

a. Requiere cirugía en las próximas 24 horas b. Las probabilidades de que la vesícula esté perforado son superiores a lo usual c. La colecistectomía laparoscópica está contraindicada d. Inicialmente la colecistitis es estéril

Page 69: Gran Compendio Final Cirugia

e. La etiología del cuadro es obstructiva 356. Una mujer de 55 años está en su cuarto día de hospitalización por un cuadro de pancretitis aguda biliar.

En la TAC de hace 24 hrs. Se aprecia necorosis pancreática extensa con gas en el retroperitoneo. El próximo paso a seguir sería:

e. Cultivo de la necrosis pancreática por punción percutánea f. Intubación endotraqueal y ventilación mecánica g. Cirugía h. Continuar el manejo conservador

357. Todas las siguientes son ciertas respecto a la pancreatitis aguda biliar excepto: a. La colecistectomía es parte fundamental del tratamiento integral b. La CPRE siempre está indicada durante el episodio agudo c. Un cálculo obstruyendo la papila es el evento desencadenante d. La TAC es el estudio de imagenología de elección 358. Ud. es llamada (o) a las 2 am de la sala de cirugía porque el paciente que se le realizó una colecistectomia

abierta a las 8 am del día anterior se queja de mucho dolor. Su enfoque de Dx y manejo inicial incluye todas las siguientes excepto:

a. Historia Clínica y Exámen Físico b. R-X de Tórax c. Hemograma completo d. Urinálisis

Page 70: Gran Compendio Final Cirugia

359. Varón de 75 años de edad es traído al servicio de urgencias por su familia para evaluación por ictericia. El paciente se queja de prurito de 2 semanas de evolución y pérdida de peso reciente de 5 Kg. A la exploración física se encuentra ictericia intensa y tiene una masa globosa en el cuadrante superior derecho del abdomen que se mueve con la respiración. El diagnostico más probable que tenga este paciente es:

a. Cirrosis hepática. b. Hepatitis viral. c. Neoplasia de la cabeza de páncreas. d. Neoplasia de la cola del páncreas 360. Todas las siguientes son ciertas respecto a la pancreatitis aguda biliar excepto: a. La colecistectomia es parte fundamental del tratamiento integral b. La CPRE siempre está indicada durante el episodio agudo c. Un cálculo obstruyendo la papila es el evento desencadenante d. La TAC es el estudio de imagenología de elección 361. Son funciones de la colecistoquinina excepto: a. Retrasa el vaciamiento gástrico. b. Estimula la contracción de la vesícula biliar. c. Relaja el esfínter anal promoviendo la defecación.. d. Estimula la secreción pancreática. e. Juega un papel importante en la regulación de la saciedad

362. Un hombre de 59 años consulta por ictericia y prurito. El US muestra las vías biliares en 9 mm (Normal <

de 6 mm) y colelitiasis. Todos los siguientes serían adecuados en el manejo inicial de este paciente excepto. a. Colangio resonancia b. ERCP c. Laparotomía d. Perfil de coagulación e. TAC de Abdomen. 363. En el manejo inicial de la pancreatitis severa están indicados todos, excepto:

a. Antibióticos de ampli espectro b. TAC en las primeras 48 horas c. Atención en la unidad de cuidados intensivos d. Laparotomía por necrosis peri-pancreáticas e. Monitoreo hemodinámica invasivo

Piel 364. Indicaciones para la excisión de un nevo pigmentado incluyen:

a. Cambio en el color, tamaño y forma b. Desarrollo de dolor de la lesión c. La aparición de lesiones satélites en el aérea de un nevo previamente existente d. Adenopatía regional inexplicable e. Todas las anteriores

365. Todos los siguientes enunciados con respecto a las quemaduras electricas son ciertas excepto:

Page 71: Gran Compendio Final Cirugia

a. Ellas son usualmente mas profundas y mas severas que lo que aparentan b. Los vasos sanguíneos los nervios y los huesos tienen una resistencia baja a la corriente eléctrica c. La necrosis muscular puede estar sobreestimada d. Todos los pacientes con lesiones eléctricas necesitan monitoreo continuo e. El manitol puede ser utilizado para mantener una diuresis adecuada para prevenir el daño renal secundaria a la mioglobinuria Pareo

a. Bupivacaína b. Lidocaína c. Ambas d. Ninguna

B Dosis máxima de infiltración 300 mg A Dosis máxima de infiltración 175 mg B Efecto dura hasta dos horas A Efecto dura hasta ocho horas B La dosis máxima de infiltración se puede incrementar si se usa adrenalina

a. Secretina b. Colecistoquinina c. Ambas d. Ninguna

___Secreción de gránulos de Zimógeno ___Secretada en el duodeno ___Secreción de agua y electrolitos ___Secreción disminuida por la administración de inhibidores de la bomba de protones ___Secreción disminuida por resección pancreático duodenal

a. Carcinoma lobular in situ b. Carcinoma ductal in situ c. Ambos d. Ninguno

B Rara vez es palpable A Nunca es palpable A Es un factor de riesgo C Puede avanzar a carcinoma invasor B Lumpectomía sin vaciamiento axilar A Observación A Mastectomía profiláctica D Quimioterapia pre-operatoria B quimioterapia post-operatoria B Microcalcificaciones

Para Futuras Generaciones 24 de Marzo de 2008